You are on page 1of 39

1 (181 SCRA 443) FERNANDEZ, Pauline August Momongan

FACTS::
Sunbeam Convenience Foods, Inc. is the recipient of a Sales Patent issued by the Bureau of Lands over two
parcels of land in Bataan. An OCT was thereby issued. The Solicitor-General filed an action for reversion on the
ground that the lots were forest lands and therefore inalienable.
CA ruled, upholding the Solicitor-General's contention.

ISSUE:
 Whether or not land is alienable and disposable land of the State.
HELD:
The SC affirmed.
Our adherence to the Regalian Doctrine subjects all agricultural, timber, and mineral lands to the dominion of
the State. Thus, before any land may be declassified from the forest group and converted into alienable or
disposable land for agricultural purposes, there must be a positive act from the Government. Even rules on the
confirmation of imperfect titles do not apply unless and until the land classified as forest land is released in an
official proclamation to that effect so that it may form part of the disposable agricultural lands of the public
domain.
The mere fact that a title was issued by the Director of Lands does not confer any validity on such title if the
property covered by the title or patent is part of the public forest.

2 Republic vs Sayo (191 SCRA 71) TACDER, Lovella Fe Madelo


FACTS:: The case at bar started at 1961 whe the spouses Casiano and Luz Sandoval filed an application for a
parcel of land, Lot 7454 originally party of Santiago but had since then been transferred to Nueva Vizcaya.

The registration was opposed by Bayaua, Reyes, and the Philippine Cacao and Farm Products.

The case went on until on 1981, 20 years after, the Heirs of Sandoval, Heirs of Bayaua, and the Bureau of
Lands and Bureau of Forest Development entered a compromise agreement, which effectively distributed
parts of lot 7454 among the aforesaid parties and the counsel of the Heirs of Sandoval as attorney's fees. The
compromise agreement was approved by the court and confirmed the title and ownership of the parties in
accordance with its terms.

Having knowledge of the incident, the Solicitor General filed a complaint before the court to annul the
decision rendered by the court a quo for being void and made in excess of jurisdiction or with grave abuse of
discretion.

The Solicitor General contended that the the Heirs of Sandoval et. al. did not present any evidence to support
their claims of ownership or registration, nor did the government agencies involve have a y authority to enter
into the compromise agreement, and finally, that he was not notified of the proceedings and so had not
opportunity to take part therein.

As for the Heirs of Sandoval et.al.'s contention, they asseverate that the land is not a public land as the
possessory information title in their name and of their predecessors-in-interest, the pre-war certification
appearing in the Bureau of Archives, and the fact that the proceeding of the registration was brought under
the Torrens act which presupposes an existing title to be confirmed, are all evidences that the land is a private
land.

ISSUE: W/N the respondent's evidences can be considered as proof that the lot 7454 is a private land.

RULING: NO. Under the Regalian Doctrine 2 all lands not otherwise appearing to be clearly within private
ownership are presumed to belong to the State. Hence it is that all applicants in land registration proceedings
have the burden of overcoming the presumption that the land thus sought to be registered forms part of the
public domain. 3 Unless the applicant succeeds in showing by clear and convincing evidence that the property
involved was acquired by him or his ancestors either by composition title from the Spanish Government or by
possessory information title, or any other means for the proper acquisition of public lands, the property must
be held to be part of the public domain . 4 The applicant must present competent and persuasive proof to
substantiate his claim; he may not rely on general statements, or mere conclusions of law other than factual
evidence of possession and title. 5
In the proceeding at bar, it appears that the principal document relied upon and presented by the applicants
for registration, to prove the private character of the large tract of land subject of their application, was a
photocopy of a certification of the National Library. But, as this Court has already had occasion to rule, that
Spanish document cannot be considered a title to property, it not being one of the grants made during the
Spanish regime, and obviously not constituting primary evidence of ownership. 6 It is an inefficacious
document on which to base any finding of the private character of the land in question.

It thus appears that the decision of the Registration Court a quo is based solely on the compromise agreement
of the parties. But that compromise agreement included private persons who had not adduced any competent
evidence of their ownership over the land subject of the registration proceeding. Portions of the land in
controversy were assigned to persons or entities who had presented nothing whatever to prove their
ownership of any part of the land. The assent of the Directors of Lands and Forest Development to the
compromise agreement did not and could not supply the absence of evidence of title required of the private
respondents.

As to the informacion posesoria invoked by the private respondents, it should be pointed out that under the
Spanish Mortgage Law, it was considered a mode of acquiring title to public lands, subject to two (2)
conditions: first, the inscription thereof in the Registry of Property, and second, actual, public, adverse, and
uninterrupted possession of the land for twenty (20) years (later reduced to ten [10] years); but where, as
here, proof of fulfillment of these conditions is absent, the informacion posesoria cannot be considered as
anything more than prima facie evidence of possession. 7

Finally, it was error to disregard the Solicitor General in the execution of the compromise agreement and its
submission to the Court for approval. It is, after all, the Solicitor General, who is the principal counsel of the
Government; this is the reason for our holding that "Court orders and decisions sent to the fiscal, acting as
agent of the Solicitor General in land registration cases, are not binding until they are actually received by the
Solicitor General."

3 Seville vs National Development Company (GR No. 129401, February 2, 2001) PLAZA, Mariafe
Manatad
Seville vs. National Development Company

FACTS::
- LSBDA (Leyte Sub-A Basin Development Authority) was created by virtue of PD no. 625. In connection
with this, a letter of instruction was executed authorizing LSBDA to purchase or acquire privately owned land.
- In June 14, 1980, certain Calixto Yap sold to LSBDA a 464,920 square meter parcel of land. The LSBDA
was able to register it under its name through the approved miscellaneous sales application with the Bureau
of Lands.
- However in May 11, 1990, petitioner Seville filed a complaint for recovery or real property, rentals and
damages against LSBDA, contending that he acquired the property and its predecessor-in-interest for more
than 30 years and therefore they are the rightful owner of it.
- The RTC rendered judgment in favor of the petitioner, when the case reached in the CA, the latter
reversed the RTC decision on the ground that petitioner’s possession of the said property was never ripen into
ownership since it still remains under the inalienable land of public domain.

ISSUE:
- W/N LSBDA ‘s title to property valid?
- W/N petitioner’s claim of ownership over the property was correct?

RULING:
- The SC ruled in favor of LSBDA. According to the SC, Seville or the Ortega estate did not become the
owner of the subject property even though they possessed the land for more than 30 years, because at the
time they occupied the said land, it is still part of inalienable land of public domain. Until the State declares it
to be alienable and disposable, no matter how long they possessed it, it may not ripen into ownership.
- Here the petitioner failed to discharge the burden of proving in court that the subject property is
already outside the component of inalienable land of public domain when they occupied the same.
- LDBDA is a rightful owner of the said land because, it acquired ownership and title over it through Sales
patent application which was approved by the Bureau of Land, this shows that the said property is no longer
under the ambit of inalienable public land.
4 Director of Lands vs Intermediate Appellate Court (219 SCRA 108) PELAEZ, Chester Bryan Pepito
FACTS::
The land involved is an island known as Tambac Island in Lingayen Gulf situated in the Municipality of Bani,
Pangasinan. Pacific Farms, Inc. initially applied for registration the subject property under the Land
Registration Act (Act No. 496). The Republic of the Philippines, thru the Director of Lands opposed the
application alleging that Pacific Farms, Inc. does not possess a fee simple title to the land nor did its
predecessors possess the land for at least thirty (30) years immediately preceding the filing of application. The
opposition likewise specifically alleged that the applicant is a private corporation disqualified under the (1973)
new Philippine Constitution from acquiring alienable lands of the public domain citing Section 11, Article 14.
The Director of Forest Development also entered its opposition alleging that the land is within the unclassified
public land and, hence, inalienable. Other private parties also filed their oppositions, but were subsequently
withdrawn. Pacific Farms, Inc. filed a manifestation-motion to change the applicant from Pacific Farms, Inc. to
J. Antonio Araneta.
Issue:
WON the land known as the "Tambac Island" can be subject of registration.
Held:
No. The Court held that lands of the public domain are classified under three main categories, namely:
Mineral, Forest and Disposable or Alienable Lands. Under the Commonwealth Constitution, only agricultural
lands were allowed to be alienated. Their disposition was provided for under Commonwealth Act No. 141
(Secs. 6-7), which states that it is only the President, upon the recommendation of the proper department
head, who has the authority to classify the lands of the public domain into alienable or disposable, timber and
mineral lands. Mineral and Timber or forest lands are not subject to private ownership unless they are first
reclassified as agricultural lands and so released for alienation. In the absence of such classification, the land
remains as unclassified land until released therefrom and rendered open to disposition. Courts have no
authority to do so.
This is in consonance with the Regalian doctrine that all lands of the public domain belong to the State, and
that the State is the source of any asserted right to ownership in land and charged with the conservation of
such patrimony. Under the Regalian Doctrine, all lands not otherwise appearing to be clearly within private
ownership are presumed to belong to the State. Hence, a positive act of the government is needed to
declassify a forest land into alienable or disposable land for agricultural or other purposes.
The burden of proof in overcoming the presumption of state ownership of the lands of the public domain is on
the person applying for registration that the land subject of the application is alienable or disposable.
Since the subject property is still unclassified, whatever possession the applicant may have had and however
long, cannot ripen into private ownership. The conversion of subject property does not automatically render
the property as alienable and disposable.
5 United Paracale vs Dela Rosa (221 SCRA 108) PASAOL, Ric Jason Patlingrao
6 Republic vs Register of Deeds of Quezon (224 SCRA 537) PAMISARAN, Excel Joy G.
FACTS::

Petitioner was awarded a 17-hectare parcel of land, by virtue of which he was issued an OCT.

Through an investigation conducted by the Bureau of Lands, it was found that the free patent acquired by
Petitioner was fraudulent. A case for falsification of public documents was filed by Petitioner was acquitted of
the crime.

Subsequently, the Solicitor-General filed a complaint against Petitioner, praying for the declaration of nullity of
the Free Patent and the OCT.

Petitioner's main contention was that the land in question was no longer within the unclassified public forest
land because by the approval of his application for Free Patent by the Bureau of Lands, the land was already
alienable and disposable public agricultural land. He also claimed that the land was a small portion of Lot
5139, an area which had been declared disposable public land by the cadastral court.

ISSUE:
Whether or not the land is alienable and disposable public land

HELD:

Under the Regalian Doctrine, all lands not otherwise clearly appearing to be privately-owned are presumed to
belong to the State. Forest lands, like mineral or timber lands which are public lands, are not subject to private
ownership unless they under the Constitution become private properties. In the absence of such classification,
the land remains unclassified public land until released therefrom and rendered open to disposition.

The task of administering and disposing lands of the public domain belongs to the Director of Lands, and
ultimately the Secretary of Agriculture and Natural Resources. Classification of public lands is, thus, an
exclusive prerogative of the Executive Department, through the Office of the President. Courts have no
authority to do so.

Thus, in controversies involving the disposition of public agricultural lands, the burden of overcoming the
presumption of state ownership of lands of the public domain lies upon the private claimant.

In the present case, Petitioner failed to present clear, positive and absolute evidence to overcome said
presumption and to support his claim.

Moreover, the fact the Petitioner acquired a title to the land is of no moment, notwithstanding the
indefeasibility of title issued under the Torrens System. The indefeasibility of a certificate of title cannot be
invoked by one who procured the same by means of fraud.

7 Ituralde vs Falcasantos (GR No. 128017, January 20, 1999) NALLA, Glene Alacayde
FACTS::
On October 17, 1986, Ramon Ituralde, herein petitioner, acquired by purchase from the heirs of Pedro Mana-
ay a parcel of land, with an area of 6.0000 hectares, more or less. However, on November 3, 1986, respondent
Alfredo Falcasantos applied with the Bureau of Lands for the award to him of the same parcel of land under
free patent, which application was protested by Ramon Ituralde.

On February 7, 1989, the Regional Director of Lands rendered a decision giving respondent a period of one
hundred twenty (120) days to exercise the right to repurchase the land by reimbursing petitioner of all
expenses he incurred in the purchase of the property in question, and held in abeyance respondent's
application for free patent. Alfredo failed to exercise the right of repurchase, which amounted to a waiver. The
Director of Land then rejected Alfredo’s application and allowed Ramon Ituralde to file a public land
application for the subject land. Meanwhile, in a case for the recovery of ownership and possession filed by
Ituralde against Alfredo, the trial court rendered a decision declaring petitioner the owner and the possessor
of the subject parcel of land with all the improvements existing thereon, with an area of 3.1248 hectares, and
ordering respondent to vacate the land in question. Ramon appealed to the decision. However the Court of
Appeals reversed the decision of the trial court and dismissed Roman’s appeal. Hence this case.

Issue: WON the Court of Appeals committed an error in reversing the decision of the trial court and dismissing
petitioner’s appeal.

Ruling: NO.
Petitioner argued that the Court of Appeals erred in setting aside the trial court's decision in his favor and
dismissing the complaint because when the Director of Lands allowed petitioner to file a public land
application for said property, it was equivalent to a declaration that said land was no longer part
of the public domain.
We deny the petition. The Court of Appeals correctly held that "the evidence is unrebutted that the subject
land is within the Forest Reserve Area as per L.C. Map No. 1557 certified on August 13, 1951'." and, hence,
not capable of private appropriation and occupation.
In Republic vs. Register of Deeds of Quezon, we held that "Forest land, like mineral timber lands which are
public lands, are not subject to private ownership unless they under the Constitution, become private
properties. In the absence of such classification, the land remains unclassified public land until released
therefrom and rendered open to disposition."
In Sunbeam Convenience Foods Inc. vs. Court of Appeals, we said: "Thus, before any land may be
declassified from the forest group and converted into alienable or disposable land for agricultural or other
purposes, there must be a positive act from the government. Even rules on the confirmation of imperfect
titles do not apply unless and until the land classified as forest land is released in an official proclamation to
that effect so that it may form part of the disposable agricultural lands of the public domain."
Hence, a positive act of the government is needed to declassify a forest land into alienable or disposable land
for agricultural or other purposes. "
And the rule is "Possession of forest lands, however long, cannot ripen into private ownership."
What is more, there is yet no award or grant to petitioner of the land in question by free patent or other ways
of acquisition of public land. Consequently, he cannot lawfully claim to be the owner of the land in question.

8 Republic vs. Court of Appeals and dela Rosa (GR No. L-43938, 04/15/1988) MISTERIO, John Kessler
Sumauang
FACTS::

The case is about a parcel of land whose ownership is being disputed by four parties: the Dela Rosas, Benguet
Consolidated Inc. (BCI), Atok Corp. (Atok) and the Bureau of Forestry Development (BFD).

Sometime in 1965, Jose dela Rosa, on his own behalf and on behalf of this three children, filed an application
for the registration of a parcel of land divided into 9 lots in Benguet. According to the children, they had
acquired the land by virtue of prescription. As evidenced, they produced tax declarations and realty tax
receipts.

BCI and Atok opposed their application claiming that the lots were covered by mineral claim sold to it in
1934. BCI, on lots 1-5, contented that it had been in actual, continuous and exclusive possession of the land in
the concept of an owner. As evidence, BCI presented geological maps, payments of taxes and construction on
the land.

Atok claims of ownership on a portion of lots 1-5 and all of lots 6-9 is similar to that of BCI, that a mineral claim
covering that the lots had been sold to them in 1931 and produced evidence such as the construction and tax
payments.

BFD objected that the land in dispute was covered by the Central Cordillera Forest Reserve under Proc. No.
217. And since it was a forest land, it cannot be subject to alienation, pursuant to the 1935 and 1973
Constitution.

The Lower Court denied the application, while the CA reversed the decision, affirming the surface rights of the
Dela Rosas, while also the sub-surface rights of the BCI and Atok. In other words, the CA ruled that the surface
was to be agricultural and mineral underneath.

ISSUE:
1. Who has better claim on the land in dispute
2. WON land can be classified as both agricultural and mineral at the same time.

HELD:

1ST ISSUE:

The Court held that BCI and Atok have vested rights over the land. The Court found that the mineral claims of
both BCI and Atok have been perfected prior to the approval of the 1935 Constitution. As a legal effect of a
valid mineral claim, it segregates the area from the public domain and confers to the locator the beneficial
ownership of the claim. As of 1935, they were removed from the public domain and had become private
properties of BCI and Atok. Even if the land was included in the Central Cordillera forest reserve, it did not
impair the rights vested in both mining companies. The claim of the Dela Rosas was disregarded for weak
evidence, and even so, they could not have acquired the land through prescription since the lad had already
been converted to mineral land.

2ND ISSUE:
No. Land cannot be classified as half agricultural and half mineral. The Court held that the classification of land
must be categorical: the land must either completely be mineral or agricultural. In this case, while the land
was initially classified as forest land, it ceases to be so and completely became mineral when the mining claims
were perfected. Even if the surface was being tilled by the Dela Rosas, it is still to be considered as mineral
land.

As long as mining operations were being undertaken thereon or underneath, it is still mineral, and not
agricultural.

Hence, the Court ruled that BCI and Atok have exclusive rights to the property in question by virtue of their
respective mining claims which they validly acquired before the 1935 Constitution which prohibits the
alienation of the lands of the public domain except agricultural lands. It could not have been transferred to
private respondents by virtue of acquisitive prescription not could its use be shared simultaneously by them
and the mining companies for agricultural and mineral purposes.

Wherefore, the decision of the CA was set aside, and reinstated the decision of the lower court.

9 Heirs of Gozo v. Philippine Union Mission Corp. of the Seventh Day Adventist Church, G.R. No. 195990,
[August 5, 2015]) LOZADA, Leah Amaya
FACTS::

Petitioners are heirs of Spouses Rafael and Concepcion Gozo who possessed a property in Brgy. Lala,
Kolambugan, Lanao del Norte of 236,638 sq.m.

Respondent (PUMC-SDA) claimed that a portion of that property was donated to it by spouses Gozo based on
the deed of donation dated 28 Feb. 1937.

When spouses Gozo donated the property in 1937, they were not yet the registered owners of the property
although they are the lawful possessors thereof.

In 1953, the land was registered under the name of spouses Gozo and an OCT was issued in their favor
pursuant to Homestead Patent granted by the President of the Phil.

After Rafael Gozo’s dealth, his wife Concepcion and their 6 children, herein petitioners, caused the
extrajudicial partition of the property. Thus, the Register of Deeds issued a TCT under the names of the
heirs/petitioners in 1954.

When Concepcion caused the survey and the subdivision, it was at this point that respondent brought to the
attention of Concepcion that 5,000 sq.m. portion of the property is already owned by it
based on the deed of donation. Upon learning from the ROD that the donation was not annotated in the title,
petitioners refused to recognize the donation. After Concepcion’s death, the heirs/petitioners continued to
pursue their claims to recover the subject property from respondent.

In 2000 or after 6 decades after the alleged deed of donation was executed, petitioners filed an action for
declaration of nullity of deed of donation and recovery of possession and ownership against
respondent. Petitioners argued that the signatures of their parents in the deed is not genuine and the
donation lack the formality of acceptance, hence invalid.

Respondent argued that since 1937 they are in open, continuous, exclusive and adverse possession of the
property in the concept of an owner, that they have built therein a church and an elementary school and that
the petitioners’ claim of lack of acceptance 63 years after the deed was executed is already barred by laches.

In 2004, the RTC rendered a decision in favor of petitioners declaring that since the deed is void for lack of
acceptance, they are the rightful owners thereof. The Court of Appeals reversed the RTC decision on the
ground of laches stating that petitioners failed to assert their rights over the land for
more than 60 years, thus laches had set in.

ISSUE:
Whether or not the donor is necessarily the owner of the real estate donated?

RULING:

No. The donors may be the possessor at the time of the donation in 1937, but they are not the owners
thereof as the subject property was part of the public domain. It was only in 1953 that ownership of the
property was vested by the State to the spouses Gozo by virtue of its issuance of the OCT pursuant to the
Homestead Patent granted by the President of the Phil. Hence the donor is not necessarily the owner of the
real property they donated when the said property was still part of the public domain (inalienable). The
donors cannot give what they do not have, nemo dat quod non habet.

-ooo-
10 Lee Hong Kok vs. David (GR No.G.R. No. L-3038 12/27/1972)
LARANAS, Rascille Mae Dagoon
Nature:
Lee Hong Kok, et al filed an appeal on certiorari seeking to reverse the decision of the Court of Appeals which
affirmed the decision of the lower court in dismissing the complaint to have the Torrens Title of Aniano David
be declared null and void.
FACTS::
 Aniano David acquired lawful title to a parcel of land pursuant to his miscellaneous sales application.
An order of award and for issuance of a sales patent was made by the Director of Lands on June 18,
1958, covering Lot 2892 containing an area of 226 square meters, which is a portion of Lot 2863 of the
Naga Cadastre.
 On the basis of the order of award of the Director of Lands, the Undersecretary of Agriculture and
Natural Resources issued Miscellaneous Sales Patent No. V-1209 pursuant to which an OCT was issued
by the Register of Deeds of Naga City to Aniano David.
 Since the filing of the sales application of Aniano David and during all the proceedings in connection
with said application, up to the actual issuance of the sales patent in his favor, Lee Hong Kok did not
put up any opposition or adverse claim to Lot 2892.
 The opposition was fatal because after the registration and issuance of the certificate and duplicate
certificate of title based on a public land patent, the land covered thereby automatically comes under
the operation of RA 496 subject to all the safeguards provided therein. Under Section 38 of RA 496,
any question concerning the validity of the certificate of title based on fraud should be raised within 1
year from the date of the issuance of the patent otherwise the certificate of title
becomes indefeasible after the lapse of 1 year.
 The contention of Lee Hong Kok was that David’s lot is a private property for it was formed thru the
process of accretion.
Issue:
Can the patent certificate of Aniano David to the property be nullified?
Held:
No. There is no legal justification for nullifying the right to the disputed lot arising from the grant made in his
favor.
1. The lot in question is NOT a private property as the Director of Lands and the Secretary of Agriculture
and Natural Resources have always sustained its public character for having been formed by
reclamation and not accretion.
1. RECLAMATION is the act of filling submerged land by deliberate acts and reclaiming title
thereto.
2. ACCRETION - is the process whereby the soil is deposited; accretion resulting from the
gradual deposit by or sedimentation from the waters belongs to the owners of the land
bordering on streams, torrents, lakes, or rivers
1. Therefore the only remedy available to the appellants is an action for reconveyance
on the ground of fraud.
2. But did he commit any fraud?
1. NO. Aniano David has not committed any fraud in applying for the purchase of
the Lot because everything was done in the open. The notices regarding the
auction sale of the land were published, the actual sale and award thereof to
Aniano David were not clandestine but open and public official acts of an officer
of the Government. The application was merely a renewal of his deceased wife's
application who had occupied the land since 1938.
2. If the grant was presumed to be invalid, who has the right to question it?
3. Only the Government, represented by the Director of Lands or the Secretary of Agriculture
and Natural Resources can bring an action to cancel a void certificate of title pursuant to a
void patent.
4. The private parties cannot claim that the patent and title issued for the land involved are void
since they are not the registered owners thereof nor had they been declared as owners in the
cadastral proceedings of the Naga Cadastre after claiming it as their private property.
5. Citing Maninang vs Consolacion: ‘The fact that the grant was made by the government is
undisputed. Whether the grant was in conformity with the law or not is a question which the
government may raise, but until it is raised by the government and set aside, the defendant
cannot question it. The legality of the grant is a question between the grantee and the
government.’
3. By what authority does the government have in disposing of the land in question?
6. IMPERIUM refers to the government authority possessed by the state which is appropriately
embraced in the concept of sovereignty. DOMINIUM, on the other hand, is appropriate with
reference to lands held by the state in its proprietary character. In such capacity, it may provide
for the exploitation and use of lands and other natural resources, including their disposition,
except as limited by the Constitution.
7. The manifestation of the concept of jura regalia, which was adopted by the present
Constitution, was embodied in the universal feudal theory that all lands were held from the
Crown, the ownership however is vested in the state rather than the head thereof.
8. As to the unappropriated public lands constituting the public domain, the sole power of
legislation is vested in Congress.
9. There being no evidence whatever that the property in question was ever acquired by the
applicants or their ancestors either by composition title from the Spanish Government or by
possessory information title or by any other means for the acquisition of public lands, the
property must be held to be public domain (Heirs of Datu Pendatun v. Director of Lands).
10. “No public land can be acquired by private persons without any grant, express or implied,
from the government.” Therefore it is indispensable that there be a showing of a title from the
state or any other mode of acquisition recognized by law otherwise the property is and remains
part of the public domain.
4. Was David’s title to the property already indefeasible?

11. As far back as 1919, in Aquino v. Director of Lands, 29 Justice Malcolm, speaking for the Court,
stated: "The proceedings under the Land Registration Law and under the provisions of
Chapter VI of the Public Land Law are the same in that both are against the whole world, both
take the nature of judicial proceedings, and for both the decree of registration issued is
conclusive and final."
12. Meanwhile in Cabacug v. Lao. There is this revealing excerpt appearing in that decision: "It is
said, and with reason, that a holder of a land acquired under a free patent is more favorably
situated than that of an owner of registered property. Not only does a free patent have a force
and effect of a Torrens Title, but in addition the person to whom it is granted has likewise in his
favor the right to repurchase within a period of five years." 33 It is quite apparent, therefore,
that petitioners' stand is legally indefensible.
RULING:
The decision of the CA of January 31, 1969 and its resolution of March 14, 1969 are affirmed.
IMPORTANT CONCEPTS
IMPERIUM DOMINIUM

The State’s authority to govern is embraced in 1. The capacity of the State to own and
the concept of sovereignty that includes acquire property. It covers such rights as
passing laws concerning a territory, maintaining title to land, exploitation and use of it,
peace and order over it, and defending it and disposition or sale of the same.
against foreign invasion. 2. It refers to lands held by the
government in a proprietary character:
can provide for the exploitation and use
of lands and other natural resources.

When the State acts in this capacity, jure The Regalian Doctrine whereby all lands of the
imperii, it generally enjoys state immunity. public domain belong to the State, and anyone
claiming title has the burden to show
ownership, comes within this concept. In this
capacity, jure gestium, the State descends to
the status of ordinary persons and thus
becomes liable as such.
11 Cruz vs Secretary of DENR (GR No. 135385, 12/6/2000) IMPIG
12 Cariñovs Insular Government 41 Phil. 935 (1909) INOK, Erick Jay Noro
FACTS::
Mateo Carino(appellant) filed his petition in the Court of Land Registration to be granted a parcel of land
consisting of 40 Hectares,1are and 13 Centares in Baguio, Province of Benguet together with a house erected
thereon . It was granted , but the Government of the Philippines and also on behalf of the United States
averred having taken possession of the property for public and military purposes, Respondents also asserted
that they had title to all the land in the Philippines except to permit private lands to be acquired and no
prescription runs against the Spanish crown. The US succeeded the title of Spain through Treaty of Paris and
Mateo’s land was not registered and that he had lost all his rights and now is deemed to be a mere
trespasser. Then the Court of First Instance dismissed the application for Mateo since he did not possessed
the land since time immemorial and the land was property of the Government. The decision was affirmed by
the Philippine Supreme Court. Thus the case was brought to the US Supreme Court by virtue of Writ of
Error(general method of bringing cases to this court, an appeal the exception, confined to equity in the main.

Mateo Carino in his appeal stated that he is an Igorot of the Province of Benguet, they have owned the land
for more than 50 years before the Treaty of Paris ,they all had been recognized as owners by the Igorots and
had inherited or received his land from his father in accordance with Igorot custom. However, no document of
title had been issued from the Spanish Crown
Issue : WON Carino owns the land.
Held : Yes
The US Supreme Court reversed the decision of the Philippine Supreme Court.
Mateo Carino should be granted what he seeks and should not be deprived of what by the practice and belief
of those among whom he lived, was his property, through a refined interpretation of an almost forgotten law
of Spain.
The grant to the plaintiff was the result of the principle of Prescription as mentioned in the royal cedula of
1754 states : “ Where such possessors shall not be able to produce title deeds, it shall be sufficient if they s
hall show that ancient possession, as a valid title by prescription” . Moreover, the Decree of June 25, 1880
states that possessors for certain time shall be deemed owners, if a cultivated land 20 years, if uncultivated 30
years.
Mateo’s father was the owner of the land by the very terms of this decree- by Organic Act of July 1, 1902 ,all
the property and rights acquired there by the United States are to be administered “for the benefit of the
inhibitants thereof.”
13 Acting Registrars of Land Titles and Deeds of Pasay City vs. Regional Trial Court Branch 57 Makati (GR
No. 81564, 4/26/1990) HONTANOSAS, Luzviminda Nee Corong
G.R. No. 81564 April 26, 1990
ACTING REGISTRARS OF LAND TITLES AND DEEDS OF PASAY CITY, PASIG AND MAKATI, METRO MANILA,
petitioners,
vs.
THE REGIONAL TRIAL COURT, BRANCH 57, IN MAKATI, METRO MANILA PRESIDED OVER BY THE HONORABLE
JUDGE FRANCISCO X. VELEZ, AND THE INTESTATE ESTATE OF THE LATE DELFIN CASAL, represented by
DOMINGO C. PALOMARES, ADMINISTRATOR, respondents.
FACTS::
On November 5, 1985, Domingo Palomares, as administrator of the heirs of Delfin Casal, commenced
suit with the Regional Trial Court, Branch 132, Makati, Metro Manila for declaratory relief, quieting of title,
cancellation of Transfer Certificate of Title No. 192, and cancellation of entries upon Original Certificate of Title
No. 291.
However, during the trial the court found hard evidence on record that: (1) the property covered by
OCT No. 291 had been conveyed to the United States of America; (2) it had been later ceded to the Republic of
the Philippines; and (3) as a consequence, OCT No. 291 was cancelled upon final orders of Judge Ostrand.
During the pendency of the case also, Proclamation No. 192 (“RESERVING FOR THE VETERANS CENTER
SITE PURPOSES CERTAIN PARCEL OF LAND OF THE PUBLIC DOMAIN SITUATED IN THE PROVINCE OF RIZAL,
ISLAND OF LUZON”) and Proclamation No. 423 (“RESERVING FOR MILITARY PURPOSES CERTAIN PARCELS OF
THE PUBLIC DOMAIN SITUATED IN THE MUNICIPALITY OF PASIG, TAGUIG, AND PARAÑAQUE, PROVINCE OF
RIZAL, AND PASAY CITY”) were issued by the government.
ISSUE:
Whether or not, Proclamation 192 and 423 were valid in the absence of proof of ownership of the
property by the government?
HELD:
Yes. Such proclamations have the character of official assertions of ownership, and the presumption
is that they have been issued by right of sovereignty and in the exercise of the State’s dominical authority. We
take not only judicial notice thereof but accept the same as a valid asseveration of regalian right over
property.

14 Palomo v. Court of Appeals, 266 SCRA 392 TAN, Cesnee Joyce V.


Palomo v. CA (266 SCRA 392, G.R. No. 130906)

FACTS::
In 1913, some 440,530 square meters of land in Albay were reserved for provincial park purposes by virtue of
EO No. 40. Of said area, 15 parcels of land were registered in the name of Diego Palomo by the Court of First
Instance.

In 1937, Diego Palomo donated these lands to his heirs Ignacio and Carmen Palomo. Claiming that the
aforesaid original certificates of title were lost during the Japanese occupation, Ignacio Palomo filed a petition
for reconstitution with the Court of First Instance of Albay on May 30, 1950. The Register of Deeds of Albay
issued Transfer Certificates of Title Nos. 3911, 3912, 3913 and 3914 sometime in October 1953.

On July 10, 1954, President Magsaysay issued Proclamation No. 47 converting the area covered by EO 40 into
the Tiwi Hot Spring National Park. The Palomos contended that they have been in possession of the subject
lands and have introduced improvements thereon.

ISSUE: Were the Original Certificate of Titles issued to the petitioners valid? –NO
HELD
Before the Treaty of Paris in 1899, the lands, whether agricultural, mineral, or forest were under the exclusive
patrimony and dominion of the Spanish crown.Private ownership of land could only be acquired through royal
concessions which were documented in various forms, such as (1) Titulo Real or Royal Grant," (2) Concession
Especial or Special Grant, (3) Titulo de Compra or Title by Purchase and (4) Informacion Posesoria or
Possessory Information title obtained under the Spanish Mortgage Law or under the Royal Decree of January
26, 1889.
The decision of the CFI relied upon by petitioners were not signed by the judge but were merely certified
copies of notification to Diego Palomo bearing the signature of the Clerk of Court.

More importantly, the lands in question were not classified as alienable lands. Since the lands were made part
of a reservation for provincial park purposes, they form part of the forest zone. Thus, they cannot be the valid
subject of alienation.

15 Republic vs Candy Maker Inc (GR No. 163766, June 22, 2006) RAÑON, Rexie Monicimpo
FACTS::
On April, 29, 1999, Antonia, Eladia, and Felisa, all surnamed Cruz, executed a Deed of Absolute Sale in favor of
Candy Maker, Inc. for a parcel of land located below the reglementary lake elevation of 12.50m, about 900
meters away the Laguna de Bay. Candy Maker, Inc. as applicant, filed an application with the MTC of Taytay,
Rizal for registration of its alleged title over the lot. The CENRO of Antipolo City declared the land to fall within
the alienable and disposable zone. On the other hand, the Land Registration Authority recommended the
exclusion of lot no. 3138-B on the ground that it is a legal easement and intended for public use, hence,
inalienable and indisposable. On July 2001, the Republic of the Philippines, the Laguna Lake Development
Authority (LLDA) filed its opposition which alleged that the lot subject of the application for registration may
not be alienated and disposed since it is considered part of the Laguna Lake Bed, a public land within, its
jurisdiction.

ISSUE:
Whether the property subject of the amended application is alienable and disposable property of the State,
and if so, whether respondent adduced the requisite quantum of evidence to prove its ownership over the
property?

HELD:
The property subject of this application was alienable and disposable public agricultural land. However,
respondent failed to prove that it possesses registrable title over the property. The statute of limitations with
regard to public agricultural lands does not operate against the statute unless the occupant proves possession
and occupation of the same after a claim of ownership for the required number of years to constitute a grant
from the State. A mere casual cultivation of portions of the land by the claimant does not constitute sufficient
basis for a claim of ownership, such possession is not exclusive and notorious as to give rise to presumptive
grant from the state. In light of the foregoing, the petition of the Republic of the Philippines is granted.
16 Chavez vs Public Estates Authority [G.R. No. 133250. July 9, 2002] GURO, Farhana
17 Alba vs Court of Appeals (GR No. 120066, 9/9/1999) ENERO, Jomari Ivan Tagud

GR 120066, September 9, 1999

FACTS::
In 1958, Lachica filed an application for title to a 4,485 sq.m. parcel of land which he had acquired through
purchase dating back to 1945. However, some parties filed their opposition thereto invoking that they are
owners of some parts of land within the total area applied for. The lower court ruled in favor of Lachica and
held, among others, that Lachica had been in actual, open and continuous possession of the subject land in
the concept of owner since 1945 and thus had acquired it through prescription.

ISSUE:
Did Lachica acquire the subject property by prescription?

RULING:
The Court ruled on the negative.

Based from the FACTS: and evidences presented, it was proven that Lachicha only had a title to a 620sq.m.
portion of the total area. Prescription cannot be had on the remaining area as the Court held that the law
applicable in this case is Sec. 48 of CA 141 (wc deals with registration of lands of public domain) and not Sec.
19 of Act 496 (wc deals with registration of private lands) and with which the lower courts had relied on.

The law in force at the time an action accrues is what governs the proceeding consistent with the fundamental
dictum that laws shall have no retroactive effect, unless the contrary is proved. In this case, the lower courts
relied on the provisions on prescription with the assumption that the subject property is a private land.
However, the application for registration should be that of a judicial confirmation of an imperfect title
considering that the land is presumed under the Regalian Doctrine to be part of the public domain.

Public lands are classified into (1) alienable or disposable lands which includes agricultural lands and (2)
inalienable or non-disposable lands or those not susceptible of private appropriation which includes Timber
lands and Mineral lands. For purposes of administration and disposition, the lands of the public domain
classified as "disposable" or "alienable" are further sub-classified into a.] Agricultural; b.] Residential,
commercial, industrial or for similar productive purposes; c.] Educational, charitable or other similar purposes,
and d.] Reservations for town sites and for public and quasi-public purposes.

Possession of public agricultural land, however long the period may have extended, never confers title thereto
upon the possessor and it is because the statute of limitations with regard to public agricultural land does not
operate against the State, unless the occupant can prove possession and occupation of the same under claim
of ownership for the required number of years to constitute a grant from the State.
Under (b), Sec. 48, CA 141, confirmation of an imperfect title to a public domain requires that:
1. There be an open, continuous, exclusive and notorious possession and occupation of agricultural lands
of the public domain;
2. It should be under a bona fide claim of ownership; and
3. possession should be for at least thirty years immediately preceding the filing of the application for
confirmation of title except when prevented by war or force majeure

In this case, Lachica had not yet satisfied the requirement of the 30 years possession, hence, prescription
cannot be granted in favor of him. JOM

18 Republic vs Imperial (GR No. 130906, 2/11/1990) DALISAY, Armando, Jr. D


Case Digest: Republic vs. Imperial
FACTS::
On September 12, 1917, the late Elias Imperial was issued Original Certificate of Title (OCT) 408 (500)
pursuant to Decree No. 55173 of then Court of First Instance of Albay. OCT No. 55173 was subdivided and
further subdivided resulting in the issuance of several titles, which are now the subjects of herein petition in
the name of private respondents. Petitioner Republic of the Philippines filed a case with the trial court to
judicially declare the Transfer Certificates of Title (TCT) issued to herein private respondents null and void
on the ground that the subject land, on which the OCT was based, has the features of a foreshore land
based on an investigation conducted by the DENR, Region V, Legazpi City. Respondents, on the other hand
contend that Director of Lands found Jose Baritua's land covered by TCT No.18655, which stemmed from
OCT 408(500), to be "definitely outside of the foreshore area."
Within the time for pleading, private respondents EANCRA Corporation, Lolita Alcazar and Salvador Alcazar
filed their answer with cross-claim, while the rest, namely, Felix S. Imperial, Feliza S. Imperial, Elias S.
Imperial and Miriam S. Imperial filed a motion to dismiss. They contended that the adjudication by the
cadastral court is binding against the whole world including the Republic since the cadastral proceedings are
in rem and the government itself through the Director of Lands instituted the proceedings and was a direct
and active participant therein. Petitioner, through the Office of the Solicitor General, filed an objection to
the motion to dismiss. After hearing the motion to dismiss, the trial court dismissed the complaint on the
ground that the judgment rendered by the cadastral court in G.R. Cad. Rec. No. 88 and the Courts resolution
in the petition to quiet title, G.R. 85770, both decreed that the parcel of land covered by OCT No. 408 (500)
was not foreshore. Petitioner appealed to the Court of Appeals. The appellate court denied petitioners
motion for reconsideration for lack of merit and for failure to file the appellants brief within the extended
period granted to petitioner.
Hence, the present petition.
ISSUE: Whether or not the petition should be granted.
HELD: Yes.
At the core of the controversy is whether the parcels of land in question are foreshore lands. Foreshore land
is a part of the alienable land of the public domain and may be disposed of only by lease and not otherwise.
It was defined as "that part (of the land) which is between high and low water and left dry by the flux and
reflux of the tides." It is also known as "a strip of land that lies between the high and low water marks and,
is alternatively wet and dry according to the flow of the tide."
The classification of public lands is a function of the executive branch of government, specifically the
director of lands (now the director of the Lands Management Bureau). The decision of the director of lands
when approved by the Secretary of the Department of Environment and Natural Resources (DENR) as to
questions of fact is conclusive upon the court.
There is allegedly a conflict between the findings of the Director of Lands and the DENR, Region V, in the
present case. Respondents contend that the Director of Lands found Jose Baritua's land covered by TCT
No.18655, which stemmed from OCT 408(500), to be "definitely outside of the foreshore area." Petitioner,
on the other hand, claims that subsequent investigation of the DENR, Region V, Legazpi City, disclosed that
the land covered by OCT No. 408 (500) from whence the titles were derived "has the features of a foreshore
land." The contradictory views of the Director of Lands and the DENR, Region V, Legazpi City, on the true
nature of the land, which contradiction was neither discussed nor resolved by the RTC, cannot be the
premise of any conclusive classification of the land involved.
The need, therefore, to determine once and for all whether the lands subject of petitioner's reversion
efforts are foreshore lands constitutes good and sufficient cause for relaxing procedural rules and granting
the third and fourth motions for extension to file appellant's brief. Petitioner's appeal presents an
exceptional circumstance impressed with public interest and must then be given due course.
In the case at bar, the need to determine once and for all whether the lands subject of petitioners reversion
efforts are foreshore lands constitutes good and sufficient cause for relaxing the procedural rules and
granting the third and fourth motions for extensions to file appellants brief. Petitioner Republics appeal
presented an exceptional circumstance impressed with public interest which in the Courts discretion must
be given due course.

19 Director of Lands vs Court of Appeals and Bisnar (GR No. 83609, 10/26/1989) CLARK, Immaculate
Gonzales
Director of Lands v. Court of Appeals (178 SCRA 708)
FACTS::
On July 20,1976, Ibarra and Amelia Bisnar filed their joint application for the registration of two parcels of
land, located in the province of Capiz, in the CFI of Capiz. They claimed that they inherited those parcels of
land. The Director of Lands and Director of the Bureau of Forest Development opposed the application on the
ground that said parcels of land were part of a timberland, a public dominion, so it cannot be the subject of
the registration proceedings. After the hearing, the CFI ordered the registration of the title of the lots in the
names of the applicants, herein private respondents after finding that the applicants and their predecessors-
in-interest have been in open, public, continuous, peaceful and adverse possession of the subject parcels of
land under bona fide claims of ownership for more than 80 years. The CA affirmed the CFI’s decision, holding
that the classification of the lotsas timberland by the Director of Forestry cannot prevail in the absence of
proof that the said lots are indeed more valuable as forest land than as agricultural land, citing as authority the
case of Ankron vs. Government of the Philippine Islands (40 Phil.10).
Issue/s:
Whether or not the possession of forestlands or timberlands for 80 years can ripen to private ownership.
Ruling:
No. The Court ruled that possession of forestlands, however long, cannot ripe ninto private ownership. It
emphasized that a positive act of the government, particularly the Executive Department is needed to
declassify land, which is classified as forest, and to convert it into alienable or disposable land for agricultural
or other purposes before registration of which may proceed. The Court, citing various cases, stated that a
parcel of forestland is within the exclusive jurisdiction of the Bureau of Forestry, an office under the Executive
Department, and beyond the power and jurisdiction of the cadastral court to register under the Torrens
System.
20 Montano vs Insular Government (GR No. 3714, 1//26/1909) CEBALLOS, Jesus Cadavez
FACTS::

Montano applied for land registration of a parcel of land used for fishery in Libis, Caloocan. This was
opposed to by the Solicitor-General (Sol. Gen.) contending that the land is owned by the US Government as
well as Obras Pias de la Sagrda Mitra as it was the absolute owner of the lot.

The Court of Land Registration granted the application of Montano. Only the Sol. Gen. appealed the
decision.

Issue:

Whether the lands used as a fishery , for the growth of nipa, and as salt deposits, inland some distance
from the sea, and asserted, though not clearly proved to be overflowed at high tide could be registered as
private property.

Held:

Yes, it can.

The Supreme Court held that since fish ponds are not classified as forest or mineral lands, it must
necessarily be agricultural land. The Court noted that before Montano owned the lot, bacawan had been
sown and propagated in the mud by the owner.
The Court also noted that government land is NOT similar to public domain or public land.
Government land includes not only public domain, but also other lands of the Government already reserved or
developed to public use or subject to private right.
Government owns real estate which is part of public lands and other real estate which is not part
thereof. There is much real property belonging to the Government which is not affected by statutes for the
settlement, prescription or sale of public lands. Examples in point are properties occupied by public buildings
or devoted to municipal or other governmental uses.

21 Lee Hong Kokvs David (GR No.G.R. No. L-30389, 12/27/1972) CAMASURA, Jayson Ug-ay
FACTS::
§ Aniano David acquired lawful title pursuant to his miscellaneous sales application in accordance with
which an order of award and for issuance of a sales patent (*similar to public auction) was made by the
Director of Lands on June 18, 1958, covering Lot 2892.

§ On the basis of the order of award of the Director of Lands the Undersecretary of Agriculture and
Natural Resources issued on August 26, 1959, Miscellaneous Sales Patent No. V-1209 pursuant to which
OCT No. 510 was issued by the Register of Deeds of Naga City on October 21, 1959.

§ Land in question is not a private property as the Director of Lands and the Secretary of Agriculture and
Natural Resources have always sustained the public character for having been formed by reclamation (as
opposed to peittioners contention that it is accretion)

§ The only remedy: action for reconveyance on the ground of fraud - But there was no fraud in this case

ISSUES:
1. W/N Lee Hong Kok can question the grant. - NO

2. W/N David has original acquisition of title. - YES

HELD: Court of Appeals Affirmed. (no legal justification for nullifying the right of David to the disputed lot
arising from the grant made in his favor by respondent officials)
§ Only the Government, represented by the Director of Lands, or the Secretary of Agriculture and Natural
Resources, can bring an action to cancel a void certificate of title issued pursuant to a void patent. The
legality of the grant is a question between the grantee and the government. Private parties like the
plaintiffs cannot claim that the patent and title issued for the land involved are void since they are not the
registered owners thereof nor had they been declared as owners in the cadastral proceedings of Naga
Cadastre after claiming it as their private property.

§ Well-settled Rule : no public land can be acquired by private persons without any grant, express or
implied, from the government

§ Cabacug v. Lao: holder of a land acquired under a free patent is more favorably situated than that of an
owner of registered property. Not only does a free patent have a force and effect of a Torrens Title, but in
addition the person to whom it is granted has likewise in his favor the right to repurchase within a period
of 5 years.

§ Imperium v. Dominium

1. Imperium - government authority possessed by the state which is appropriately embraced in the
concept of sovereignty

2. Dominium - capacity to own or acquire property. The use of this term is appropriate with
reference to lands held by the state in its proprietary character. In such capacity, it may provide for the
exploitation and use of lands and other natural resources, including their disposition, except as limited by
the Constitution.

22 Director of Lands vs CA and Valeriano (GR No. 58867, 6/22/1984) CABALLERO, Jeremiah Napalan
23 Menguito vs Republic (GR No. 134308, 12/14/2000) BENITEZ, Winnie L
Menguito v. Republic
[G.R. No. 134308. December 14, 2000]

FACTS::

A petition for review assailing the Court of Appeals Sept 30, 1997 decision against the Menguito’s –the
petitioners, with promulgated resolution 10-Mos. later reversing the decision of the Regional Trial Court of
Pasig City. The RTC decision confirmed the application for the titling of the parcel of land with aggregate area
of 2112 sqm located at Brgy Ususan, Taguig, Metro Manila, in favor of the Menguito’s.
The application of TCT was brought by the operation of the Land Registration of Authority as amended by the
Property Registration Decree No. 1529 proceeding to declare:

1. That its applicants –the Menguito’s, are owners, in fee simple, these 11 parcels of land.
2. Listing the applicants lot title numbers, attendant documents given with their respective Technical
Descriptions.

For the said application, the RTC of Pasig issued a notice of its initial hearing against the whole world
publishing the same at Abante Tabloid on April 5, 1989.

Seven days earlier, however, March 30, 1989, the Office of Solicitor General, filed its contention as:
1. Applicants nor its predecessor’s in interest were neither in open, continuous, exclusive,
adverse and notorious possession or occupation of the land they applied for since 1945.
2. Applicants don’t have competent and sufficient evidence of bonafide acquisition –without
open, uninterrupted-continuous, exclusive, adverse or notorious occupation of the lot in the
concept of the owner and so appearing not genuine or indicative of pretended possession.
3. Titling from fee simple with Spanish grant title were not anymore available after Feb 16,
1976 as required by PD 892.
4. That the said parcel of land applied for is part of the public domain and belonging to the
Republic of the Philippines –and not subject to private appropriation.

The OSG thus stated its valid opposition on the presented documents by the applicants leaving its prayers that
said application be denied and land in question be reverted to the ownership of the Republic of the
Philippines.

At the appellate court, the RTC’s decision favoring the registrations of the land applied were reversed and thus
this petition.

ISSUE: WON the CA’s did not err in its decision to reverse the trial court findings.

HELD: No. The petition is without merit.

The Court cited Sec. 48 of Commonwealth Act No. 141 as amended, provide the registration the
registration of the title of lands in this wise;

“(b) Who by themselves, or their predecessors in interest have been in open, continuous, exclusive, and
notorious occupation of the land in the concept of the owner.”

The Court thus observed that the petitioners have insufficient evidence on this.”

The Court likewise pointed that;

• Applicants to avail the OCT they must overcome the presumption that the land they applied for
forms part of the public domain and if so, it has to be shown as re-classified or alienated to a private
person by the state.
• Otherwise, the land remain inalienable public domain.
• To overcome this, an incontrovertible or clear and convincing evidence must be presented.
• The court observed that applicants possession were tacked only as far back as 1948 to its
predecessors of interest –the same must have shown that their predecessors in interest were in
possession of the property by some 30-years back or 1938 before WW-2.

The court find the petitioners failed to show it even as they claimed that Cirilo Menguito once
declared the land for tax purposes in 1943 –yet failed to show documentary evidence to support the claim.

The court therefore find no reason to modify the appellate court’s decision -thus denied the
petition and declared cost against petitioner.

Note:

Menguito v. Republic:

“Unless public land is shown to have been reclassified or alienated to a private person by the State, it remains
part of the inalienable public domain. Indeed, occupation thereof in the concept of owner, no matter how
long, cannot ripen into ownership and be registered as a title.

24 MesinavsSonza (GR No. L-14722, 5/25/1960) BANTUGAN


25 Susi vsRazon (G.R. No. L-24066December 9, 1925) BALUCANAG, April Gem Balansag
FACTS::
On September 5, 1899, Valentin Susi bought a parcel of land from Apolonio Garcia and Basilio Mendoza. Prior
to such purchase, the said sellers have been in an open, continuous, adverse and public possession of the land
for 19 years.
After the purchase, Valentin Susi took possession of the land in an open, continuous, adverse and public
manner and without interruption. However on September 13, 1913, the possession was interrupted when
Angela Razon filed an action to recover possession of the land. The Court of First Instance dismissed Angela
Razon’s complaint.
Despite the dismissal of the case, Angela Razon filed for the purchase of the disputed land with the Director of
Lands. Valentin Susi filed his opposition and asserted his right over the land. However, the Director of Land
overruled the opposition and sold the land the Angela Razon on the ground that the land was still and public
land and Valentin Susi does not have a title over it.

ISSUE:
WON Valentin Susi already acquired title to the land.

HELD:
YES. Valentin Susi has been in possession of the land in question openly, continuously, adversely and publicly
personally and through his predecessors for 45 years. This being the case, the doctrine laid down by the
Supreme Court of United States in Cariño vs Government of the Philippine Islands is applicable. Here, it held
that To acquire a right to a certificate of title over a land of the public domain, under the provisions of Chapter
VI of Act No. 926, as amended by Chapter VIII of Act No. 2874, an open, adverse, public and continuous
possession from July 26,1894, is sufficient, provided the possessor makes application therefor under the
provisions of section 47 of Act No. 2874. The possessor under such circumstances acquires by operation of
law, not only a right to a grant, but a grant of the government, and the actual issuance of a title is not
necessary in order that said grant may be sanctioned by the courts.
In the present case all the requirements for a grant were complied with since Valentin Susi has been in
possession of the land in question openly, continuously, adversely, and publicly, personally and through his
predecessors, since the year 1880, that is, for about forty-five years. By a legal fiction, Valentin Susi had
acquired the land in question by a grant of the State, it had already ceased to be the public domain and had
become private property, at least by presumption, of Valentin Susi, beyond the control of the Director of
Lands. Consequently, in selling the land in question to Angela Razon, the Director of Lands disposed of a land
over which he had no longer any title or control, and the sale thus made was void and of no effect, and Angela
Razon did not thereby acquire any right.
26 Alba vs Court of Appeals (GR No. 120066, 9/9/1999) APURADA, Kathryne Vencio
FACTS::
Applicant Jose Lachica filed this application for title to land on April 28, 1958 with the claim that the land
applied for was purchased by him and his wife, Adela Raz from one Eulalio Raz. The documents attached to
the application are: technical description, surveyor's certificate, certification by the chief deputy assessor of
Aklan and the blue print of Psu-161277. The land applied for is residential.
The initial hearing was held on and an order of general default was issued but those who presented their
opposition, namely, Octabela Alba Vda. De Raz, Manuel and Susana Braulio, Jose Rago, representing Apolonia
Rebeco.
Petitioners contended that they were the real owners of the subject land, which they have been possession
continuously, openly and peacefully under claim of ownership for not less than 70 years together with their
predecessor-in-interest, and the applicant is only entitled to have the portion of 620 square meters which is
contrary to his application of 4,845 square meters.
The trial court finds that Dr. Jose Lachica as the absolute owner in fee simple of the land described in his
application for its original registration in his name.

Issue:
WON the private respondent/applicant is entitled to the confirmation of his ownership in fee simple for the 4,
845 square meter parcel of land he applied for.

Held: NO
In Section 48 of Commonwealth Act 141, (which is the law enforced during the filing of application) as
amended by RA Nos. 1942 and 6236, which states that:
Sec. 48. The following-described citizens of the Philippines, occupying lands of the public domain or claiming
to own any such lands or an interest therein, but whose titles have not been perfected or completed, may
apply to the Court of First Instance of the province where the land is located for confirmation of their claim
and issuance of a certificate of title therefor, under the Land Registration Act, to wit:
(a) Those who prior to the transfer of sovereignty from Spain to the United States have applied for the
purchase, composition or other form of grant of lands of the public domain under the laws and royal decrees
then in force and have instituted and prosecuted the proceedings in connection therewith, but have with or
without default upon their part, or for any other cause, not received title therefor, if such applicants or
grantees and their heirs have occupied and cultivated said lands continuously since the filing of their
applications. 49
(b) Those who by themselves or through their predecessors in interest have been in open, continuous,
exclusive and notorious possession and occupation of agricultural lands of the public domain under a bona
fide claim of ownership, for at least thirty years immediately preceding the filing of the application for
confirmation of title except when prevented by war or force majeure. These shall be conclusively presumed to
have performed all the conditions essential to a Government grant and shall be entitled to a certificate of title
under the provisions of this chapter. 50
(c) Members of the national cultural minorities who by themselves or through their predecessors-in-interest
have been in open, continuous, exclusive and notorious possession and occupation of lands of the public
domain suitable to agriculture, whether disposable or not, under a bona fide claim of ownership for at least 30
years shall be entitled to the rights granted in subsection (b) hereof. 51
A circumspect scrutiny of the assailed Decision readily shows that in the affirming the ruling of the trial court,
the Court of Appeals relied on the provisions of Section 19 of Act 496 52 in relation to the Civil Code's
provision's on prescription on the assumption that the subject land is private land. Therein lies the flaw in the
appellate court's postulate. The application for registration of private respondent is for judicial confirmation of
an imperfect title considering that the land is presumed under the Regalian Doctrine to be part of the public
domain.
The private respondent failed to satisfy the Supreme Court that he complied the condition set forth for the
judicial confirmation of his title for the following reasons, to wit;
a. as to the (a) of Section 48 of CA 141- respondent did present any imperfect title with Spanish decree.
b. as to the (b) of Section 48 of CA 141- prescription cannot be applied as there were no express grant from
the government that the subject land was amongst the agricultural land certified as alienable and
disposable. Ultimately, No public land can be acquired by private persons without any grant, express or
implied from the government; it is indispensable that there be a showing of title from the state.
c. nor he is members as specified on the (c) condition of CA 141.

27 Republic vs Court of Appeals and Naguit (GR No. 1/17/2005) ANTOPINA, Babielen Poliquit
FACTS::
On January 5, 1993, Naguit filed a petition for registration of title of a parcel of land. The application sought a
judicial confirmation of imperfect title over the land.

The public prosecutor, appearing for the government, and Angeles opposed the petition. The court issued an
order of general default against the whole world except as to Angeles and the government.

The evidence revealed that the subject parcel of land was originally declared for taxation purposes in the
name of Urbano in 1945. Urbano executed a Deed of Quitclaim in favor of the heirs of Maming, wherein he
renounced all his rights to the subject property and confirmed the sale made by his father to Maming
sometime in 1955 or 1956. Subsequently, the heirs of Maming executed a deed of absolute sale in favor of
respondent Naguit who thereupon started occupying the same.

Naguit constituted Blanco, Jr. as her attorney-in-fact and administrator. The administrator introduced
improvements, planted trees in addition to existing coconut trees which were then 50 to 60 years old, and
paid the corresponding taxes due on the subject land.

Naguit and her predecessors-in-interest had occupied the land openly and in the concept of owner without
any objection from any private person or even the government until she filed her application for registration.

The OSG argued that the property which is in open, continuous and exclusive possession must first be
alienable. Since the subject land was declared alienable only on October 15, 1980, Naguit could not have
maintained a bona fide claim of ownership since June 12, 1945, as required by Section 14 of the Property
Registration Decree, since prior to 1980, the land was not alienable or disposable.

The OSG suggested an interpretation that all lands of the public domain which were not declared alienable or
disposable before June 12, 1945 would not be susceptible to original registration, no matter the length of
unchallenged possession by the occupant.
Issue:

Whether or not it is necessary under Section 14(1) of the Property Registration Decree that the subject land be
first classified as alienable and disposable before the applicant’s possession under a bona fide claim of
ownership could even start.

Held:

Section 14 of the Property Registration Decree, governing original registration proceedings, provides:

SECTION 14. Who may apply.— The following persons may file in the proper Court of First Instance an
application for registration of title to land, whether personally or through their duly authorized
representatives:

(1) those who by themselves or through their predecessors-in-interest have been in open, continuous,
exclusive and notorious possession and occupation of alienable and disposable lands of the public domain
under a bona fide claim of ownership since June 12, 1945, or earlier.

(2) Those who have acquired ownership over private lands by prescription under the provisions of existing
laws.

There are three obvious requisites for the filing of an application for registration of title under Section 14(1) –
that the property in question is alienable and disposable land of the public domain; that the applicants by
themselves or through their predecessors-in-interest have been in open, continuous, exclusive and notorious
possession and occupation, and; that such possession is under a bona fide claim of ownership since June 12,
1945 or earlier.

The OSG's interpretation would render paragraph (1) of Section 14 virtually inoperative and even precludes
the government from giving it effect even as it decides to reclassify public agricultural lands as alienable and
disposable. The unreasonableness of the situation would even be aggravated considering that before June 12,
1945, the Philippines was not yet even considered an independent state.

The more reasonable interpretation of Section 14(1) is that it merely requires the property sought to be
registered as already alienable and disposable at the time the application for registration of title is filed. If the
State, at the time the application is made, has not yet deemed it proper to release the property for alienation
or disposition, the presumption is that the government is still reserving the right to utilize the property; hence,
the need to preserve its ownership in the State irrespective of the length of adverse possession even if in good
faith. However, if the property has already been classified as alienable and disposable, as it is in this case, then
there is already an intention on the part of the State to abdicate its exclusive prerogative over the property.

In this case, the 3 requisites for the filing of registration of title under Section 14(1) had been met by
Naguit. The parcel of land had been declared alienable; Naguit and her predecessors-in-interest had been in
open, continuous, exclusive and notorious possession and occupation of the land evidenced by the 50 to 60-
year old trees at the time she purchased the property; as well as the tax declarations executed by the original
owner Urbano in 1954, which strengthened one's bona fide claim of ownership.
28 Heirs of Malabanan v. Republic, G.R. No. 179987, [April 29, 2009], 605 PHIL 244-
326) JESURA
FACTS::

On 20 February 1998, Mario Malabanan filed an application for land registration before the
RTC of Cavite-Tagaytay, covering a parcel of land situated in Silang Cavite, consisting of
71,324 square meters. Malabanan claimed that he had purchased the property from
Eduardo Velazco, and that he and his predecessors-in-interest had been in open,
notorious, and continuous adverse and peaceful possession of the land for more than thirty
(30) years. Velazco testified that the property was originally belonged to a twenty-two
hectare property owned by his great-grandfather, Lino Velazco. Lino had four sons–
Benedicto, Gregorio, Eduardo and Esteban–the fourth being Aristedes’s grandfather. Upon
Lino’s death, his four sons inherited the property and divided it among themselves. But by
1966, Esteban’s wife, Magdalena, had become the administrator of all the properties
inherited by the Velazco sons from their father, Lino. After the death of Esteban and
Magdalena, their son Virgilio succeeded them in administering the properties, including Lot
9864-A, which originally belonged to his uncle, Eduardo Velazco. It was this property that
was sold by Eduardo Velazco to Malabanan.
Among the evidence presented by Malabanan during trial was a Certification dated 11 June
2001, issued by the Community Environment & Natural Resources Office, Department of
Environment and Natural Resources (CENRO-DENR), which stated that the subject
property was “verified to be within the Alienable or Disposable land per Land Classification
Map No. 3013 established under Project No. 20-A and approved as such under FAO 4-
1656 on March 15, 1982.” On 3 December 2002, the RTC approved the application for
registration.

The Republic interposed an appeal to the Court of Appeals, arguing that Malabanan had
failed to prove that the property belonged to the alienable and disposable land of the public
domain, and that the RTC had erred in finding that he had been in possession of the
property in the manner and for the length of time required by law for confirmation of
imperfect title. On 23 February 2007, the Court of Appeals reversed the RTC ruling and
dismissed the application of Malabanan.
ISSUES:
1. In order that an alienable and disposable land of the public domain may be registered
under Section 14(1) of Presidential Decree No. 1529, otherwise known as the Property
Registration Decree, should the land be classified as alienable and disposable as of June
12, 1945 or is it sufficient that such classification occur at any time prior to the filing of the
applicant for registration provided that it is established that the applicant has been in open, continuous,
exclusive and notorious possession of the land under a bona fide claim of
ownership since June 12, 1945 or earlier?
2. For purposes of Section 14(2) of the Property Registration Decree may a parcel of land
classified as alienable and disposable be deemed private land and therefore susceptible to
acquisition by prescription in accordance with the Civil Code?
HELD:

The Petition is denied.


(1) In connection with Section 14(1) of the Property Registration Decree, Section 48(b) of
the Public Land Act recognizes and confirms that “those who by themselves or through
their predecessors in interest have been in open, continuous, exclusive, and notorious
possession and occupation of alienable and disposable lands of the public domain, under a
bona fide claim of acquisition of ownership, since June 12, 1945” have acquired ownership
of, and registrable title to, such lands based on the length and quality of their possession.

(a) Since Section 48(b) merely requires possession since 12 June 1945 and does not
require that the lands should have been alienable and disposable during the entire period of
possession, the possessor is entitled to secure judicial confirmation of his title thereto as
soon as it is declared alienable and disposable, subject to the timeframe imposed by
Section 47 of the Public Land Act.
(b) The right to register granted under Section 48(b) of the Public Land Act is further
confirmed by Section 14(1) of the Property Registration Decree.

(2) In complying with Section 14(2) of the Property Registration Decree, consider that
under the Civil Code, prescription is recognized as a mode of acquiring ownership of
patrimonial property. However, public domain lands become only patrimonial property not
only with a declaration that these are alienable or disposable. There must also be an
express government manifestation that the property is already patrimonial or no longer
retained for public service or the development of national wealth, under Article 422 of the
Civil Code. And only when the property has become patrimonial can the prescriptive period
for the acquisition of property of the public dominion begin to run.

(a) Patrimonial property is private property of the government. The person acquires
ownership of patrimonial property by prescription under the Civil Code is entitled to secure
registration thereof under Section 14(2) of the Property Registration Decree.
(b) There are two kinds of prescription by which patrimonial property may be acquired, oneordinary and other
extraordinary. Under ordinary acquisitive prescription, a person acquires
ownership of a patrimonial property through possession for at least ten (10) years, in good
faith and with just title. Under extraordinary acquisitive prescription, a person’s
uninterrupted adverse possession of patrimonial property for at least thirty (30) years,
regardless of good faith or just title, ripens into ownership.

It is clear that the evidence of petitioners is insufficient to establish that Malabanan has
acquired ownership over the subject property under Section 48(b) of the Public Land Act.
There is no substantive evidence to establish that Malabanan or petitioners as his
predecessors-in-interest have been in possession of the property since 12 June 1945 or
earlier. The earliest that petitioners can date back their possession, according to their own
evidence—the Tax Declarations they presented in particular—is to the year 1948. Thus,
they cannot avail themselves of registration under Section 14(1) of the Property
Registration Decree.

Neither can petitioners properly invoke Section 14(2) as basis for registration. While the
subject property was declared as alienable or disposable in 1982, there is no competent
evidence that is no longer intended for public use service or for the development of the
national evidence, conformably with Article 422 of the Civil Code. The classification of the
subject property as alienable and disposable land of the public domain does not change its
status as property of the public dominion under Article 420(2) of the Civil Code. Thus, it is
insusceptible to acquisition by prescription.

29 Heirs of Malabanan v. Republic, G.R. No. 179987, [September 3, 2013 COSTAN


30 Palomo vs CA (GR No. 95608, 1/21/1997) TAN, Cesnee Joyce V.
Palomo v. CA (266 SCRA 392, G.R. No. 130906)

FACTS::
In 1913, some 440,530 square meters of land in Albay were reserved for provincial park purposes by virtue of
EO No. 40. Of said area, 15 parcels of land were registered in the name of Diego Palomo by the Court of First
Instance.

In 1937, Diego Palomo donated these lands to his heirs Ignacio and Carmen Palomo. Claiming that the
aforesaid original certificates of title were lost during the Japanese occupation, Ignacio Palomo filed a petition
for reconstitution with the Court of First Instance of Albay on May 30, 1950. The Register of Deeds of Albay
issued Transfer Certificates of Title Nos. 3911, 3912, 3913 and 3914 sometime in October 1953.

On July 10, 1954, President Magsaysay issued Proclamation No. 47 converting the area covered by EO 40 into
the Tiwi Hot Spring National Park. The Palomos contended that they have been in possession of the subject
lands and have introduced improvements thereon.

ISSUE: Were the Original Certificate of Titles issued to the petitioners valid? –NO
HELD
Before the Treaty of Paris in 1899, the lands, whether agricultural, mineral, or forest were under the exclusive
patrimony and dominion of the Spanish crown.Private ownership of land could only be acquired through royal
concessions which were documented in various forms, such as (1) Titulo Real or Royal Grant," (2) Concession
Especial or Special Grant, (3) Titulo de Compra or Title by Purchase and (4) Informacion Posesoria or
Possessory Information title obtained under the Spanish Mortgage Law or under the Royal Decree of January
26, 1889.
The decision of the CFI relied upon by petitioners were not signed by the judge but were merely certified
copies of notification to Diego Palomo bearing the signature of the Clerk of Court.

More importantly, the lands in question were not classified as alienable lands. Since the lands were made part
of a reservation for provincial park purposes, they form part of the forest zone. Thus, they cannot be the valid
subject of alienation.

31 Republic vs Bracewell[G.R. No. 107427. January 25, 2000] TACDER, Lovella Fe Madelo

FACTS::

The controversy involves a total of nine thousand six hundred fifty-seven (9,657) square meters of land
located in Las Piñas, Metro Manila.

In 1908, Maria Cailles, married to James Bracewell, Sr., who acquired the said parcels of land from the
Dalandan and Jimenez families of Las Piñas; after which corresponding Tax Declarations were issued in the
name of Maria Cailles.

On January 16, 1961, Maria Cailles sold the said parcels of land to her son, the petitioner, by virtue of a Deed
of Sale which was duly annotated and registered with the Registry of Deeds of Pasig, Rizal. Tax Declarations
were thereafter issued in the name of petitioner, canceling the previous Tax Declarations issued to Maria
Cailles.

On September 19, 1963, petitioner filed before the then Court of First Instance of Pasig, Rizal an action for
confirmation of imperfect title under Section 48 of Commonwealth Act No. 141.

The Director of Lands, represented by the Solicitor General, opposed petitioner's application on the grounds
that neither he nor his predecessors-in-interest possessed sufficient title to the subject land nor have they
been in open, continuous, exclusive and notorious possession and occupation of the same for at least thirty
(30) years prior to the application, and that the subject land is part of the public domain.
On May 3, 1989, the lower court issued an Order granting the application of petitioner. The Solicitor General
promptly appealed to respondent Court which, on June 29, 1992, reversed and set aside the lower court's
Order. It also denied petitioner's Motion for Reconsideration in its Resolution of September 30, 1992.

Issues:

a) Whether the failure of the petitioner to prosecute his action for an unreasonable length of time?

b) Whether the tax declarations attached to the complaint do not constitute acquisition of the lands applied
for?

Held:

The controversy is simple. On one hand, petitioner asserts his right of title to the subject land under Section 48
(b) of Commonwealth Act No. 141, having by himself and through his predecessors-in-interest been in open,
continuous, exclusive and notorious possession and occupation of the subject parcels of land, under a bona
fide claim of acquisition or ownership, since 1908. On the other hand, it is the respondents' position that since
the subject parcels of land were only classified as alienable or disposable on March 27, 1972, petitioner did
not have any title to confirm when he filed his application in 1963. Neither was the requisite thirty years
possession met.

A similar situation in the case of Reyes v. Court of Appeals, where a homestead patent issued to the
petitioners' predecessor-in-interest was cancelled on the ground that at the time it was issued, the subject
land was still part of the public domain.

In the said case, this Court ruled as follows —

Under the Regalian doctrine, all lands of the public domain belong to the State, and that the State is the
source of any asserted right to ownership in land and charged with the conservation of such patrimony. This
same doctrine also states that all lands not otherwise appearing to be clearly within private ownership are
presumed to belong to the State (Director of Lands vs. Intermediate Appellate Court, 219 SCRA 340).

Hence, the burden of proof in overcoming the presumption of State ownership of lands of the public domain is
on the person applying for registration. The applicant must show that the land subject of the application is
alienable or disposable. These petitioners failed to do.

The homestead patent was issued to petitioners' predecessor-in-interest, the subject land belong to the
inalienable and undisposable portion of the public domain. Thus, any title issued in their name by mistake or
oversight is void ab initio because at the time the homestead patent was issued to petitioners, as successors-
in-interest of the original patent applicant, the Director of Lands was not then authorized to dispose of the
same because the area was not yet classified as disposable public land. Consequently, the title issued to herein
petitioners by the Bureau of Lands is void ab initio.

Neither has petitioner shown proof that the subject Forestry Administrative Order recognizes private or
vested rights under which his case may fall. We only find on record the Indorsement of the Bureau of Forest
Development from which no indication of such exemption may be gleaned.

Having found petitioner to have no cause of action for his application for confirmation of imperfect title, we
see no need to discuss the other errors raised in this petition.
32 Republic vs Court of Appeals and Bernabe (GR No. L-40402, 3/16/1987) RAÑON, Rexie Monicimpo
FACTS::
Lot No. 622 of the Mariveles Cadastre was declared public land in a decision rendered before the last war in
Cadastral Case No. 19, LRC Cadastral Record No. 1097. On July 6, 1965 such lot was segregated from the
forest zone and released and certified by the Bureau of Forestry (BOF) as an agricultural Land for disposition
under the Public Land Act. On April 26, 1967, Respondents filed in the CFI of Bataan a petition to reopen
Cadastral Case No. 19 to perfect their rights and register their titles to said lots. They alleged that they
acquired ownership and possession of said parcels of land by purchase from the original owners thereof,
whose possession of the same including that of the herein respondents, has always been continuous, open,
active, exclusive, public, adverse and in the concept of owners for more than 30 years. The Director of
Forestry filed an opposition to the above petition but later withdrew the same upon verification of findings
that this portion of the timberland had already been released from the mass of the public forests.
Subsequently, the Acting Prov. Fiscal of Bataan, for and in behalf of the Director of Lands filed his opposition
alleging that the land is still a Public Land and as such cannot be the subject of a land registration proceeding
under Act 496. The lower court adjudicated in favor or respondent Bernabes, finding that the latter have
complied with all the terms and conditions entitling them to a grant. This decision having become final, the
Commissioner of Land Registration issued the corresponding decrees of registration. On the other hand,
petitioner DL through the Solicitor Gen. filed a petition for review of the decrees. Afterwards, he filed an
Amended Petition for Review, adding: that respondents executed simulated deeds of sale conveying portions
of the subject parcels to third parties for fictitious considerations in order to remove the same from the
coverage of Sec. 38 of Act 496, but in truth, buyers are mere dummies of petitioners; hence, not purchasers
for value. The Court of First Instance denied this petition and on appeal, the CA affirmed the questioned
decision. Petitioner’s Motion for Reconsideration having been denied for lack of merit; hence, this petition.
ISSUE:
WON the lots claimed by respondents could be legally be the subject of a juridical confirmation of Title under
Section 48 (b) of Commonwealth Act 141 as amended by Republic Act 1942.

HELD:
No. The Supreme Court ruled that Sec. 48 (b) of CA 141, as amended, applies exclusively to public
lands. Forest lands or areas covered with forests are excluded. Thus, possession of forest lands, however long
cannot ripen into private ownership. A parcel of forest land is within the exclusive jurisdiction of the Bureau
of Forestry and beyond the power and jurisdiction of the cadastral court to register under the Torrens
System. Thus, even if the reopening of the cadastral proceedings was at all possible, private respondents have
not qualified for a grant under Section 48 (b) of CA 141. They can only be credited with 1 year, 9 mos. and 20
days of possession and occupation of the lots involved, counted from July 6, 1965 when the lots involved had
been segregated from the forest zone and released by the BOF as an agricultural land for disposition under the
Public Land Act. As such, respondents and their predecessors in interest could not have possessed the lots for
the required period of 30 years as disposable agricultural land.
33 Republic v. Bautista, Jr., G.R. No. 166890, [June 28, 2016]) PLAZA, Mariafe Manatad

FACTS::
Apolonio Bautista Jr. acquired lot 17078 through succession, when his father Apolonio Sr. died in 1987. He
applied judicial confirmation of imperfect title before MTC with testimonial evidence that his father was in
possession of the said lot since 1969 and acquired it from Jardin and Villanueva, through notarized Deed of
Absolute Sale dated February 15, 1971 and May 25, 1973 respectively. The MTC granted the application and
declared him as the owner in fee simple of the said land.
The Government of the Philippines appealed before CA, contending that the testimony of Apolonio Jr. is an
hearsay which should not be given probative value. The application of judicial confirmation of imperfect title
must comply with Sec. 48(b) of CA 141. The CA affirmed the ruling of MTC, it pointed out that the Government
of the Phil. did not timely object to the evidence presented.
ISSUE: W/N the grant of judicial confirmation of imperfect title to Apolonio Jr proper?
RULING:
The Supreme Court reversed the ruling of MTC and CA. It held that the requisite period of possession must
conform with Sec 48(b) of Public Land Act as amended by RA 1942 which provides that any person who
applies for judicial confirmation, he or his predecessor in interest must have been in open, continuous,
exclusive and notorious possession and occupation of alienable and disposable land of public domain under
the bonafide claim of ownership at least 30 years since June 12, 1945 or earlier.
In the present case, the lower court relied only on the testimony of Bautista Jr. that his father acquired the
land and in possession since 1969. He failed to prove that the transferor had in open, continuous, exclusive
and notorious possession of said land for at least 30 years since June 12, 1945 or earlier. Bautista Jr. has no
personal knowledge of these FACTS:. Lack of this evidence does not give the court the right to grant a judicial
confirmation of imperfect title in favor of Bautista Jr.

34 MIAA vs Court of Appeals (GR No. 155650, 7/20/2006) PELAEZ, Chester Bryan Pepito

FACTS::
Manila International Airport Authority (MIAA) operates the Ninoy Aquino International Airport (NAIA)
Complex in Parañaque City. The Office of the Government Corporate Counsel (OGCC) issued Opinion No. 061
which states that the Local Government Code of 1991 withdrew the exemption from real estate tax granted to
MIAA. Thus, MIAA negotiated with City of Parañaque to pay the real estate tax imposed by the City. MIAA
then paid some of the real estate tax already due.
MIAA received Final Notices of Real Estate Tax Delinquency from the City of Parañaque for the taxable years
1992 to 2001 amounting to 624M. The City of Parañaque, through its City Treasurer, also issued notices of levy
and warrants of levy on the Airport Lands and Buildings. The Mayor of the City of Parañaque threatened to sell
at public auction the Airport Lands and Buildings should MIAA fail to pay the real estate tax delinquency. MIAA
thus sought a clarification of OGCC Opinion No. 061. The OGCC issued Opinion No. 147 clarifying OGCC
Opinion No. 061 and pointed out that Local Government Code requires persons exempt from real estate tax to
show proof of exemption. The OGCC opined that Section 21 of the MIAA Charter is the proof that MIAA is
exempt from real estate tax. MIAA then filed with the Court of Appeals an original petition for prohibition and
injunction, with prayer for preliminary injunction or temporary restraining order to restrain the City of
Parañaque from imposing real estate tax on, levying against, and auctioning for public sale the Airport Lands
and Buildings. However, this was dismissed due to because MIAA filed it beyond the 60-day reglementary
period. The Court of Appeals also denied MIAA's motion for reconsideration and supplemental motion for
reconsideration. Hence, MIAA filed a petition for review before the SC.
Issue:
WON the Land and Buildings of MIAA are part of the public dominion and thus cannot be the subject of levy
and auction sale.
Held:
Yes. The Court held that the land and buildings of MIAA are part of the public dominion. Since the airport is
devoted for public use, for the domestic and international travel and transportation. Even if MIAA charge fees,
this is for support of its operation and for regulation and does not change the character of the land and
buildings of MIAA as part of the public dominion.
As part of the public dominion the land and buildings of MIAA are outside the commerce of man. To subject
them to levy and public auction is contrary to public policy. Unless the President issues a proclamation
withdrawing the airport land and buildings from public use, these properties remain to be of public dominion
and are inalienable. As long as the land and buildings are for public use the ownership is with the Republic of
the Philippines.
35 Roman Catholic Bishop of Kalibovs Municipality of Buruanga (GR No. 149145,
3/31/2006) PASAOL, Ric Jason Patlingrao

36 Amunategui vs. Director of Forestry (GR No. L-27873, 11/29/1983) PAMISARAN, Excel Joy
Gemota

37 Ankronvs Government of the Philippines (GR No. 14213, 8/23/1919) NALLA, Glene Alacayde
FACTS:: This case involves an application for registration under the Torrens system of a certain parcel of
land. The only opposition of the said application which was presented by the Director of Landswas that the
land in question was the property of the Government of the United States under the control and
administration of the Government of the Philippine Islands. The lower court ordered and decreed that said
parcel of land be registered in the name of the said applicant, J. H. Ankron, subject, however, to the right of
the Government of the Philippine Islands to open a road thereon in the manner and conditions mentioned
in said decision. On appeal, appellant, contented, among others that, that portions of said land cannot be
registered in accordance with the existing Land Registration Law for the reason that they are manglares.

Issue: WON the land in question cannot registered?

Ruling:
The mere fact that a tract of land has trees upon it or has mineral within it is not of itself sufficient to
declare that one is forestry land and the other, mineral land. There must be some proof of the extent and
present or future value of the forestry and of the minerals. While, as we have just said, many definitions
have been given for "agriculture," "forestry," and "mineral" lands, and that in each case it is a question of
fact, we think it is safe to say that in order to be forestry or mineral land the proof must show that it is more
valuable for the forestry or the mineral which it contains than it is for agricultural purposes. (Sec. 7, Act No.
1148.) It is not sufficient to show that there exists some trees upon the land or that it bears some mineral.
Land may be classified as forestry or mineral today, and, by reason of the exhaustion of the timber or
mineral, be classified as agricultural land tomorrow. And vice-versa, by reason of the rapid growth of timber
or the discovery of valuable minerals, lands classified as agricultural today may be differently classified
tomorrow. Each case must be decided upon the proof in that particular case, having regard for its present or
future value for one or the other purposes. We believe, however, considering the fact that it is a matter of
public knowledge that a majority of the lands in the Philippine Islands are agricultural lands, that the courts
have a right to presume, in the absence of evidence to the contrary, that in each case the lands are
agricultural lands until the contrary is shown. Whatever the land involved in a particular land registration
case is forestry or mineral land must, therefore, be a matter of proof. Its superior value for one purpose or
the other is a question of fact to be settled by the proof in each particular case. The fact that the land is a
manglar [mangrove swamp] is not sufficient for the courts to decide whether it is agricultural, forestry, or
mineral land. It may perchance belong to one or the other of said classes of land. The Government, in the
first instance, under the provisions of Act No. 1148, may, by reservation, decide for itself what portions of
public land shall be considered forestry land, unless private interests have intervened before such
reservation is made. In the latter case, whether the land is agricultural, forestry, or mineral, is a question of
proof. Until private interests have intervened, the Government, by virtue of the terms of said Act (No.
1148), may decide for itself what portions of the "public domain" shall be set aside and reserved as forestry
or mineral land.

38 Sta. Rosa Development Corporation vs CA (GR No. 112526, 10/12/2001) MISTERIO, John Kessler
Sumauang
FACTS::

The case is a petition regarding Department of Agrarian Reform Adjudication Board’s (DARAB) order of
compulsory acquisition of petitioner’s property under the Comprehensive Agrarian Reform Program (CARP).

Petitioner Sta. Rosa Development Corporation (SRRDC), was the registered owner of two parcel of land
situated at Brgy. Casile, Cabuyao, Laguna. According to them, these lands are watersheds which provide clean
and potable (drinkable) water to the Canlubang community and that 90 light industries are located in that
area.

They were alleging respondents usurped its rights over their property thereby destroying the ecosystem. Since
the said land provides water to the residents, respondents sought an easement of a right of a way to and from
Barangay Castile, to which, by counterclaim, Sta. Rosa sought ejectment against respondents.

Respondents went to the DAR and filed a case for compulsory acquisition of the Sta. Rosa Property under the
Comprehensive Agrarian Reform Program.

Compulsory acquisition is the power of the government to acquire private rights in land without the willing
consent of its owner or occupant in order to benefit the society.

The said land was inspected by the Municipal and Agrarian Reform Officer, and upon consensus of the
authorities concerned, they decided that the said land must be placed under compulsory acquisition.

Petitioners filed an objection on the ground that:

· The area is not appropriate for agricultural purposes.


· The area was rugged in terrain with slopes 18% and above. (which falls under the exception in compulsory
acquisition of CARP)
· The occupants of the land were illegal settlers or (squatters) who by no means are entitled to the land as
beneficiaries.

ISSUE:
· Whether or not the property in question is covered by CARP despite the fact that the entire property
formed part of a watershed area prior to the enactment of R.A No. 6657
· Whether the petition of land conversion of the parcels of land may be granted?

HELD:
Watershed is one of those enumerated by CARP to be exempt from its coverage.

· Art. 67 of PD 1067 provides that Any watershed or any area of land adjacent to any surface water or
overlying any ground water may be declared by the Department of Natural resources as a protected area.
· Watersheds may be defined as an area drained by a river and its tributaries and enclosed by a boundary
or divide which separates it from adjacent watersheds.

We cannot ignore the fact that the disputed parcels of land form a vital part of an area that need to be
protected for watershed purposes. The protection of watersheds ensures an adequate supply of water for
future generations and the control of flashfloods that not only damage property but cause loss of lives.
Protection of watersheds is an intergenerational responsibility that needs to be answered now.

Although evidence of petitioners is strong, the Supreme Court opines that the area must be maintained for
watershed purposes for ecological and environmental considerations despite the 88 families who are
beneficiaries of the CARP. It is important that a larger view of the situation be taken because of the thousands
of residents downstream if the watershed will not be protected and maintained for its natural purpose.

Despite Supreme Court’s strong opinion of protection of watersheds as an intergenerational responsibility,


they, however ordered to DARAB to conduct a re-evaluation of the case since the said land falls under
exception.

39 Collado vs Court of Appeals (GR No. 107764, 10/4/2002) LOZADA, Leah Amaya

FACTS::
Petitioner Edna T. Collado filed with the land registration court an application for registration of a parcel
of land (“Lot”), situated in Antipolo Rizal. Attached to the application was a technical description, stating “this
survey is inside IN-12 Mariquina Watershed.” The Solicitor General filed oppositions to the application.
Petitioners (Edna Collado and her co-applicants) allege that they have occupied the Lot since time
immemorial. Their possession has been open, public, notorious and in the concept of owners. They paid all
real estate taxes and submitted evidence to prove that there have been 9 transfers of rights among them and
their predecessors-in-interest. RTC ruled in favor of the petitioners for having presented sufficient evidence to
establish registrable title over the property.
ISSUE:
(1) WON petitioners have registrable title over the Lot. NO.
(2) Did petitioners acquire private rights over the parcel of land prior to the issuance of EO 33? NO.
HELD:
(1) Petitioners concede that the Lot is inside the literal description of Marikina Watershed Reservation (MWR).
Their main claim over the Lot is that “all Presidential proclamations like the proclamation setting aside the
MWR are subject to private rights.” EO 33 (which established the MWR) has a saving clause that the
reservations are “subject to existing private rights, if any there be.”
Under the Regalian Doctrine, all lands not otherwise appearing to be clearly within private ownership are
presumed to belong to the State. The Spaniards first introduced the doctrine to the Philippines through the
Laws of the Indies and the Royal Cedulas, specifically, Law 14, Title 12, Book 4 of the Novisima Recopilacion de
Leyes de las Indias which laid the foundation that "all lands that were not acquired from the Government,
either by purchase or by grant, belong to the public domain." Upon the Spanish conquest of the Philippines,
ownership of all "lands, territories and possessions" in the Philippines passed to the Spanish Crown.
The Laws of the Indies were followed by the Ley Hipotecaria or the Mortgage Law of 1893. The Spanish
Mortgage Law provided for the systematic registration of titles and deeds as well as possessory claims. The
Royal Decree of 1894 or the "Maura Law" partly amended the Mortgage Law as well as the Law of the Indies.
The Maura Law was the last Spanish land law promulgated in the Philippines. It required the "adjustment" or
registration of all agricultural lands, otherwise the lands would revert to the state.
Four years later, Spain ceded to the government of the United States all rights, interests and claims over the
national territory of the Philippine Islands through the Treaty of Paris of December 10, 1898. In 1903, the
United States colonial government, through the Philippine Commission, passed Act No. 926, the first Public
Land Act, which was described as follows:
"Act No. 926, the first Public Land Act, was passed in pursuance of the provisions of the Philippine Bill of 1902.
The law governed the disposition of lands of the public domain. It prescribed rules and regulations for the
homesteading, selling and leasing of portions of the public domain of the Philippine Islands, and prescribed
the terms and conditions to enable persons to perfect their titles to public lands in the Islands. It also provided
for the "issuance of patents to certain native settlers upon public lands," for the establishment of town sites
and sale of lots therein, for the completion of imperfect titles, and for the cancellation or confirmation of
Spanish concessions and grants in the Islands." In short, the Public Land Act operated on the assumption that
title to public lands in the Name: I. Concept of Jura Regalia Natural Resources First Set_ Philippine Islands
remained in the government; and that the government’s title to public land sprung from the Treaty of Paris
and other subsequent treaties between Spain and the United States. The term "public land" referred to all
lands of the public domain whose title still remained in the government and are thrown open to private
appropriation and settlement, and excluded the patrimonial property of the government and the friar lands."
Thus, it is plain error for petitioners to argue that under the Philippine Bill of 1902 and Public Land Act No.
926, mere possession by private individuals of lands creates the legal presumption that the lands are alienable
and disposable.
Both the 1935 and 1973 Constitutions prohibited the alienation of all natural resources except agricultural
lands of the public domain. The 1987 Constitution readopted this policy. Indeed, all lands of the public domain
as well as all natural resources enumerated in the Philippine Constitution belong to the State.
Watershed Reservation is a Natural Resource: The term "natural resource" includes "not only timber, gas, oil
coal, minerals, lakes, and submerged lands, but also, features which supply a human need and contribute to
the health, welfare, and benefit of a community, and are essential to the well-being thereof and proper
enjoyment of property devoted to park and recreational purposes."
(2) An applicant must overcome the presumption that the land he is applying for is part of the public domain
and that he has an interest to warrant registration in his name arising from an imperfect title (may have been
derived from old Spanish grants or titles). In the case at bar, petitioners were unable to acquire a valid and
enforceable right or title because of the failure to complete the required period of possession (at least 30
years).
Assuming that the Lot was alienable and disposable land prior to the issuance of EO 33 in 1904, EO 33
reserved the Lot as a watershed. Since then, the Lot became non-disposable and inalienable public land. At
the time petitioners filed their application on April 25, 1985, the Lot has been reserved as a watershed under
EO 33 for 81 years prior to the filing of petitioners’ application.
40 Director of Forestry vs. Villareal (GR No. L-32266, 2/27/1989) LARANAS, Rascille Mae Dagoon
DIRECTOR OF FORESTRY v. VILLAREAL
G.R. No. L-32266 | February 27, 1989 | Cruz, J.
FACTS:: The said land consists of 178,113 square meters of mangrove swamps located in the municipality of
Sapian, Capiz. Ruperto Villareal applied for its registration on January 25, 1949, alleging that he and his
predecessors-in-interest had been in possession of the land for more than forty years. He was opposed by
several persons, including the petitioner on behalf of the Republic of the Philippines. After trial, the
application was approved by the Court of First Instance of Capiz. The decision was affirmed by the Court of
Appeals. The Director of Forestry then came to this Court in a petition for review on certiorari claiming that
the land in dispute was forestal in nature and not subject to private appropriation. He asks that the
registration be reversed. It is undisputed by the parties that the land in dispute is a mangrove land HOWEVER
the legal nature of mangrove swamps or manglares are still in contention. Director of Forestry claims that it is
forestall and is not disposable. On the other hand, Private respondents insists that it is alienable as agricultural
land.
ISSUES: Are mangrove swamps classified as public forest lands?

RULING: YES. Part of our public forest lands, they are not alienable under the Constitution or are they
considered public agricultural lands; they may be acquired under private ownership.
Mangrove swamps or manglares should be understood as comprised within the public forests of the
Philippines as defined in the aforecited Section 1820 of the Administrative Code of 1917. The legislature
having so determined, we have no authority to ignore or modify its decision, and in effect veto it, in the
exercise of our own discretion. The statutory definition remains unchanged to date and, no less noteworthy, is
accepted and invoked by the executive department. More importantly, the said provision has not been
challenged as arbitrary or unrealistic or unconstitutional assuming the requisite conditions, to justify our
judicial intervention and scrutiny. The law is thus presumed valid and so must be respected. We repeat our
statement in the Amunategui case that the classification of mangrove swamps as forest lands is descriptive of
its legal nature or status and does not have to be descriptive of what the land actually looks like. That
determination having been made and no cogent argument having been raised to annul it, we have no duty as
judges but to apply it.
It follows from all this that the land under contention being admittedly a part of the mangrove swamps of
Sapian, and for which a minor forest license had in fact been issued by the Bureau of Forestry from 1920 to
1950, it must be considered forest land. It could therefore not be the subject of the adverse possession and
consequent ownership claimed by the private respondent in support of his application for registration. To be
so, it had first to be released as forest land and reclassified as agricultural land pursuant to the certification
the Director of Forestry may issue under Section 1827 of the Revised Administrative Code.
The Respondent even showed, a survey of the land and its tax declaration to support its claim, however the
court held that the same is insufficient especially now that the land is a forest land.
WHEREFORE, the decision of the Court of Appeals is SET ASIDE and the application for registration of title of
private respondent is DISMISSED, with cost against him. This decision is immediately executory.

41 Atok-Big Wedge Mining Corporation vs Court of Appeals, GR No. 88883,


1/18/1991) INOK, Erick Jay Noro
G.R. No. 88883 January 18, 1991

FACTS::

Fredia Mineral claim of about nine (9) hectares situated in Tuding, Itogon, Benguet, was located sometime
between December 25, 1930 and December 31, 1930, a period of six (6) days, by A.I. Reynolds in accordance
with the provisions of the Act of Congress of July 1, 1902, better known as the Philippine Bill of 1902, in a so-
called Declaration of Location.

The said Declaration of Location of mineral claim was duly recorded in the Office of the Mining Recorder
sometime on January 2, 1931. Fredia mineral claim, together with other mineral claims, was sold by A.I.
Reynolds to Big Wedge Mining Company, the earlier corporate name of Atok Big Wedge Mining Company, Inc.
(Atok for short; herein petitioner) in a Deed of Sale executed on November 2, 1931. Since then petitioner Atok
has been in continuous and exclusive ownership and possession of said claim up to the present .

Atok has paid the realty taxes and occupation fees for the Fredia mineral claim. The Fredia mineral claim
together with other mineral claims owned by Atok has been declared under Tax Declaration No. 9535 and that
in view of Presidential Decree No. 1214 an application for lease was filed by Atok covering the Fredia mineral
claim.

On the other hand, private respondent Liwan Consi has a lot below the land of a certain Mr. Acay at Tuding
Slide, Itogon, Benguet. He constructed a house thereon sometime in 1964. The lot is covered by Tax
Declaration No. 9462. When he first constructed his house below the lot of Mr. Acay he was told that it was
not necessary for him to obtain a building permit as it was only a nipa hut. And no one prohibited him from
entering the land so he was constructing a house thereon. It was only in January 1984 when private
respondent Consi repaired the said house that people came to take pictures and told him that the lot belongs
to Atok. Private respondent Consi has been paying taxes on said land which his father before him had
occupied .

On January 1984, the security guards of Atok informed Feliciano Reyes, Security Officer of Atok, that a
construction was being undertaken at the area of the Fredia mineral claim by private respondent Liwan Consi.
Feliciano Reyes instructed the cashier to go and take pictures of the construction. Feliciano Reyes himself and
other security guards went to the place of the construction to verify and then to the police to report the
matter.

On March 1, 1984, Atok filed a complaint for forcible entry and detainer against Liwan Consi , which was
dismissed after due hearing by the MTC of Itogon in favor of Liwan Consi. Petitioner ATOK appealed to the
RTC of Baguio, which reversed the decision of the MTC, ordering defendant Liwan Consi to vacate the
premises of the Fredia Mineral claim, restoring possession thereof to the plaintiff Atok Big Wedge Mining
Company. Defendant Liwan Cosi was further ordered to remove and demolish the house he constructed in the
premises of the land of Fredia Mineral claim.
In a petition for review filed by Liwan Consi with the CA, the CA rendered its decision dismissing the subject
forcible entry action, and further rule in part that: Liwan Consi had a possessory right over the property which
may mature into ownership on the basis of long-term possession under the Public Land Law. Thus, it held that
both Consi and ATOK are of equal footing with regards to the subject lot, holding possessory titles to the land.
The petitioner through its long term occupancy while respondent mining firm being the claim locator and
applicant for lease on the mineral claim.

ATOK filed a motion for reconsideration, which was denied by the CA. Hence, this petition.

ISSUE:
Whether or not an individual's long term occupation of land of the public domain vests him with such rights
over the same as to defeat the rights of the owner of that claim.

HELD:

It is of no importance whether Benguet and Atok had secured a patent for as held in the Gold Creek Mining
Corporation case, for all physical purposes of ownership, the owner is not required to secure a patent as long
as he complies with the provisions of the mining laws; his possessory right, for all practical purposes of
ownership, is as good as though secured by patent (Republic v. Court of Appeals, 160 SCRA 228 [1988]).

In the case at bar, the evidence on record pointed that the petitioner Atok has faithfully complied with all the
requirements of the law regarding the maintenance of the said Fredia Mineral Claim.

The perfection of the mining claim converted the property to mineral land and under the laws then in force
removed it from the public domain. By such act, the locators acquired exclusive rights over the land, against
even the government, without need of any further act such as the purchase of the land or obtaining of a
patent over it. As the land had become the private property of the locators, they had the right to transfer the
same, as they did, to Benguet and Atok .

As in the instant petition, the record shows that the lot in question was acquired through a Deed of Sale
executed between Atok and Fredia Mineral Claim.

It is, therefore, evident that Benguet and Atok have exclusive rights to the property in question by virtue of
their respective mining claims which they validly acquired before the Constitution of 1935 prohibited the
alienation of all lands of the public domain except agricultural lands, subject to vested rights existing at the
time of its adoption. The land was not and could not have been transferred to the private respondents by
virtue of acquisitive prescription, nor could its use be shared simultaneously by them and the mining
companies for agricultural and mineral purposes (Ibid).

On the matter of possession, private respondent contends that his predecessor-in-interest has been in
possession of said lot even before the war and has in fact cultivated the same. Since the subject lot is mineral
land, private respondent's possession of the subject lot no matter how long did not confer upon him
possessory rights over the same.

Furthermore, Article 538 of the New Civil Code provides:

Art. 538. Possession as a fact cannot be recognized at the same time in two different personalities
except in the cases of co-possession. Should a question arise regarding the fact of possession, the present
possessor shall be preferred; if there are two possessors, the one longer in possession; if the dates of the
possession are the same, the one who presents a title; and if all these conditions are equal, the thing shall be
placed in judicial deposit pending determination of its possession or ownership through proper proceedings.

Since 1931 up to the present, petitioner ATOK has been in continuous and exclusive possession of the Fredia
mineral claim while private respondent's possession started only sometime in 1964 when he constructed a
house thereon. Clearly, ATOK has superior possessory rights than private respondent, Liwan Consi, the former
being "the one longer in possession."
It is therefore clear that from the legal viewpoint it was really petitioner who was in actual physical possession
of the property. Having been deprived of this possession by the private respondent, petitioner has every right
to sue for ejectment.

With this ruling enunciated by the Court, it can further be declared and held that petitioner Atok has the
exclusive right to the property in question.

42 Palomovs Court of Appeals [G.R. No. 95608. January 21, 1997] IMPIG, Sychar M

43 Republic vs Southside Homeowners (GR No. 156951, 9/22/2006) HONTANOSAS,


Luzviminda Nee Corong

44 Republic vsAlagad (G.R. No. L-66807 January 26, 1989) GURO, Farhana

45 SIAN Enterprise vs. FF Cruz (GR no. 146616, 8/31/2006) FERNANDEZ, Pauline August Momongan

46 Director of Lands vs. Roman Catholic Archbishop of Manila (GR No. 14869) ENERO,
Jomari Ivan Tagud
FACTS::
In 1913, cadastral proceedings were conducted to settle the title to a considerable tract of land in the Province
of Rizal. The Roman Catholic Archbishop of Manila (church) and other private parties were claimants of 13
cadastral lots that comprised the contested property. The lower court ruled in favor of the private claimants.
Upon appeal, the church invoked that the composition title of the church with the Spanish Government
included the subject property. The church then presented one witness and rested. The private oppositors then
called their respective witnesses to prove title by possession, and rested. The church thereafter made an offer
to present additional testimony intended to show that the possession of the private claimants had been
interrupted and that it was merely possession through the tolerance of the church. However, the counsel for
the oppositors objected to the entry of additional testimonies which was sustained.

ISSUE:
Did the lower court err in refusing the entry the church’s additional testimonies?

RULING:
The Court ruled in the affirmative.

The object of a cadastral petition is that the title to the various lots embraced in the survey may be settled and
adjudicated. It is in the nature of a proceeding in rem, promoted by the Director of Lands, somewhat akin to a
judicial inquiry and investigation leading to a judicial decree. In one sense, there is no plaintiff and there is no
defendant. In another sense, the Government is the plaintiff and all the- claimants are defendants.

As a general rule, courts should adhere to the usual rules of practice, procedure, and evidence that governs
registration proceedings. However, in registration proceedings where so many parties are involved and action
is taken quickly and abruptly, opportunity should be given to parties to submit additional corroborative
evidence in support of their claims of title, if the ends of justice so require. This case was remanded back to
the lower court with the church being allowed to admit additional testimonies in the interest of justice and
ascertainment of truth. JOM

47 AbelleravsFarol (GR No. 48480, 7/30/1943) DALISAY, Armando, Jr. D


FABIAN B. S. ABELLERA, petitioner,
vs.
MEYNARDO M. FAROL, ET AL., respondents

FACTS::
Abellera, in a previous case concerning the same real estate involved herein, sued Hermegildo Balanag and
others who are either the same parties in this case or the latter's predecessors in interest, alleging ownership
of the land. But his complaint was dismissed by the Court of First Instance on two grounds:
(1) prescription in favor of defendants; and
(2) the deed of donation of these lands to him had not been formally accepted according to Article 633 of the
Civil Code.
Upon appeal to this Court, the judgment of the trial court was affirmed on the second ground
aforementioned. It appears in that decision of this Court that after the perfection of the appeal, Abellera
executed a public document formally accepting the donation of the land, and presented and deed of
acceptance together with proofs of notification of acceptance to the donor, as ground for new trial.
In July of 1918, or four months after the above-mentioned decision of this Court, petitioner herein brought
another action for recovery of the land against the same defendants in the previous case.
The second suit was later dismissed by the Court of First Instance and transferred to cadastral case No. 5
which included the hacienda in question that had in the meantime been subdivided into lots.
The latter through counsel moved that Abellera's claim over the lots concerned be dismissed on the grounds
of res judicata and prescription.

Issue:
Did the cadastral court, on the ground of res judicata, have any power to entertain the motion to
dismiss Abellera's claim and bar him from presenting evidence to prove his ownership of these lots?

Ruling:
Rule 132 of the Rules of Court provides:
These rules shall not apply to land registration, cadastral and election cases, naturalization and insolvency
proceedings, and other cases not herein provided for, except by analogy or in a suppletory character and
whenever practicable and convenient.
The Rules of Court may be applied in cadastral cases when two conditions are present: (1) analogy or need to
supplement the cadastral law, and (2) practicability and convenience.
The principal aim is to settle as much as possible all disputes over land and to remove all clouds over land
titles, as far a practicable, in a community.
To attain this purpose, the cadastral court should allow all claimants ample freedom to ventilate whatever
right they may assert over real estate, permitting them, in keeping with the law of evidence, to offer proofs in
support of their allegations.
We are, therefore, of the opinion that while in a cadastral case res judicata is available to a claimant in order
to defeat the alleged rights of another claimant, nevertheless prior judgment can not set up in a motion to
dismiss.

48 Gayapanao vs. IAC (GR No. 68109, 7/17/1991) CLARK, Immaculate Gonzales
Gayapanao vs. IAC (GR No. 68109, 7/17/1991)
FACTS::
This is a petition for review on certiorari filed by Severino Gayapanao and his siblings questioning the decision
of the IAC in upholding the validity of the sale of the land between their father Constantino and their sister.
The 2 hectare land, subject of this case is part of 10 hectare homestead land registered in the name of
Constantino Gayapanao under OCT. The final order of the Director of Lands for the issuance of patent was
issued on December 10, 1937. On November 15, 1938, Constantino Gayapanao executed a private deed
entitled kasulatan ng bilihan in favor of Simeona Gayapanao and his husband.
RTC’s decision: The contract of sale between Simeona Gayapanao and her father is null and void for having
been executed with the 5 year prohibitory period provided under Section 118 of the Public Land Law
CA’s decision: It reversed the decision of the RTC and uphold the validity of the sale.
ISSUE: WON CA was correct in upholding the validity of the sale?
RULING:
No, the provision of law which prohibits the sale or encumbrance of the homestead within 5 years after the
grant is mandatory.
From the date of the approval of the application and for a term of five (5) years from and after the date of
issuance of the patent or grant, lands acquired under free patent or homestead provisions cannot be subject
to encumbrance or alienation, nor shall they become liable to the satisfaction of any debt contracted prior to
the expiration of said period. The only exception mentioned by the law is the sale or encumbrance in favor of
the government or any of its branches, units or institutions.
In a number of cases, we have consistently ruled that a sale of homestead within the five (5) year prohibitive
period is void ab initio and the same cannot be ratified nor can it acquire validity through the passage of time.

49 Republic vs Garcia (GR No. L-11597, 5/27/1959) CEBALLOS, Jesus Cadavez


FACTS::

Garcia was granted a homestead patent for his 23.21 hectares of land. After 3 years and 3 months
later (April 14, 1950), he sold 19 hectares to the lot to several persons. For this reason, the CFI of Cotabato
decreed the reversion of the property in favor of the Government for violating the selling the property within
the prohibitive period of 5 years from date of issuance under Sec. 118 of CA 141.

Garcia contends that he did not violate the said provision since the sale was not registered and that it
was not the entire land was sold.

Issue:

Whether or not Garcia violated the Sec. 118 of CA 141.

Held.

Yes, he did.

The Supreme Court held that it is enough that the property, in whole or in part, was alienated or
encumbered within the prohibitive period except in favor of the Government, Sec. 118 of CA 141 partly
provides: Except in favor of the Government or any its branches, units, or institutions, lands acquired under
free patent or homestead provision shall not be subject to encumbrance of alienation from the date of the
approval of the application and for a term of five years from and after the date of issuance of the patent or
grant, nor shall they become liable to the satisfaction of any debt contracted prior to the expiration of said
period, but the improvements or crops on the land may be mortgaged or pledged to qualified persons,
association, or corporations.

Such alienation is a sufficient cause for reversion to the State of the whole grant. In granting a
homestead to an applicant, the law imposes as a condition that the land should not be encumbered, sold or
alienated within five years from the issuance of the patent. Sec. 124 of CA 141 provides: Any acquisition,
conveyance, alienation, transfer, or other contract made or executed in violation of any of the provisions of
section118, 120, 121, 122, and 123 of this Act shall be unlawful and null and void from its execution and shall
produce the effect of annulling and cancelling the grant, title, patent, or permit originally issued, recognized or
confirmed, actually or presumptively, and cause the reversion of the property and its improvements to the
State.

50 Lee Chuy Realty Corp vs Court of Appeals (GR No. 104114, 12/4/1995) CAMASURA, Jayson Ug-
ay
FACTS::
A valuable piece of land located at Meycauyan, Bulacan, with an area of 24,576 sq. m. and covered by OCT No.
0-5290 is disputed by Lee Chuy Realty Corporation and Marc Realty and Development Corp. Such land was
originally co-owned by Ruben Jacinto(one-sixth), Dominador, Arsenio, Liwayway all surnamed Bascara and
Ernesto jacinto(collectively owned the remaining five-sixths).
On Feb. 4, 1981, Ruben Jacinto sold his one-sixth pro-indiviso share to LEE CHUY REALTY which was registered
30 April 1981. On 5 May 1989 the Bascaras and Ernesto Jacinto also sold their share to MARC REALTY which
was registered on 16 October 1989.
Lee Chuy Realty claims that it was never informed of the existence of the sale between Marc Realty and the
Bascaras/Jacinto. Marc Realty insists that Lee Chuy verbally notified of the sale and was given a copy of the
deed of sale.
On 13 November 1989 LEE CHUY REALTY filed a complaint for legal redemption against MARC REALTY and
consigned in court a manager's check for 614,400. MARC REALTY insisted that the complaint be dismissed for
failure to state a cause of action there being no allegation of prior valid tender of payment or a prior valid
notice of consignation.
On Dec 26, 1990, the trial court ruled in favour of Lee Chuy Realty which stated that there was a valid tender
of payment and consignation. It also stated that neither a separate offer to redeem nor a formal notice of
consignation is necessary for the reason that the filing of the action itself, within the period of redemption, is
equivalent to a formal offer to redeem.
On 1 February 1991 MARC REALTY filed a Petition for Certiorari, Prohibition with Temporary Restraining Order
and/or Writ of Preliminary Injunction which was referred to the Court of Appeals. The CA reversed the
decision of the lower court and ruled that "a prior tender or offer of redemption is a prerequisite or
precondition to the filing of an action for legal redemption” and that "there must be tender of the redemption
price within the required period because the policy of the law is not to leave the purchaser's title in
uncertainty beyond the established 30-day period.
MARC REALTY contends that prior tender of payment is a condition precedent to the filing of an action in court
in order to validly exercise the right of legal redemption. LEE CHUY REALTY however argues that the filing of
the action itself is equivalent to a formal offer to redeem, which is a condition precedent to the valid exercise
of the right of legal redemption.
Lee Chuy filed a motion for reconsideration but was denied

Issue:
WON a formal offer to redeem accompanied with tender of payment a condition precedent to the filing of an
action for the valid exercise of the right of legal redemption; is the filing of the action with consignation
equivalent to a formal offer to redeem

Held:
No.
In Hulganza v. Court of Appeals14 the Court, citing previous decisions, declared that the formal offer to
redeem, accompanied by a bona fide tender of the redemption price, within the prescribed period is only
essential to preserve the right of redemption for future enforcement beyond such period of redemption and
within the period prescribed for the action by the statute of limitations. Where, as in the instant case, the right
to redeem is exercised through judicial action within the reglementary period the formal offer to redeem,
accompanied by a bona fide tender of the redemption price, while proper, may be unessential. The filing of
the action itself is equivalent to a formal offer to redeem.
In sum, the formal offer to redeem is not a distinct step or condition sine qua non to the filing of the action in
Court for the valid exercise of the right of legal redemption. What constitutes a condition precedent is either a
formal offer to redeem or the filing of an action in court together with the consignation of the redemption
price within the reglementary period.
The doctrine in Tolentino, Tioseco and Belisario cases was jettisoned by the Court of Appeals on the ground
that they do not involve legal redemption by a co-owner but by a mortgagor. It concluded that the application
of the rules on legal redemption by a co-owner differs from the legal redemption by a mortgagor. But the law
does not distinguish; neither should we. For sure, the principle in the aforecited cases is applicable regardless
of whether the redemptioner is a co-owner or a mortgagor. Public policy favors redemption regardless of
whether the redemptioner is a co-owner or mortgagor, although perhaps with unequal force and effect since
each is given a fixed but different period. A co-owner desirous of exercising his right of legal redemption is
given a period of thirty (30) days from notice of the sale within which to avail of the right to redeem.15 Under
the free patent or homestead provisions of the Public Land Act a period of five (5) years from the date of
conveyance is provided,16 the five-year period to be reckoned from the date of the sale and not from the date
of registration in the office of the Register of Deeds.17 The redemption of extrajudicially foreclosed
properties, on the other hand, is exercisable within one (1) year from the date of the auction sale as provided
for in Act No. 3135.

51 Ong Ching Po vs. Court of Appeals (239 SCRA 341) CABALLERO, Jeremiah Napalan

52 Frenzel vs Catito (GR No. 143958, July 11, 2003) BENITEZ, Winnie L
ALFRED FRITZ FRENZEL, petitioner,
v.
EDERLINA P. CATITO, respondent.
[G.R. No. 143958. July 11, 2003]

CALLEJO, SR., J.:

FACTS::
Petitioner – Alfred Fritz Frenzel, is an Australian Electrical Engineer who once worked with New Guinea
Airlines, started doing business in the Philippines since 1974, and two years later got married to a Filipina
Teresita Santos. They were separated bread and board in 1981 and two years later met Ederlina Catito as a
masseuse in one of night clubs in Sydney Australia

Frenzel got enamored with Ederlina and convinced her to setup wholesome business in Manila for her own as
he was willing to help her. Ederlina returned and later Alfred followed and helped her setup a Edorial Beauty
saloon with his personal funds –let alone Ederlina as sole registered owner. Alfred went back to work in Papue
New Guinea as airline pilot while Ederlina too went to Germany to file her divorce with her husband –Muller.

When Alfred returned to the Philippines to live with Ederlina–he bought house and lot or real properties in QC
for her with his own funds and agreeing to name it under Ederlina alone accepting that as a foreigner he is not
allowed to own a real estate property under the Philippine Laws. Alfred decided to stay for good in the
Philippines and sold his personal properties in Australia with proceeds later used to buy other 2-more
properties in Davao City –again, in Ederlina’s name only. There was also an occasion when Ederlina deposited
some USD 250,000 in their joint-accounts.

Meanwhile, Ederlina’s divorce case with her German husband was denied and with the prospect that she
could be charged with bigamy along with Alfed who himself is still married. This started their common law
marriage deterioration and soon enough Alfred got penniless and filed his claim on the properties listed in
Ederlina’s name alone as well as charges the return of all other asset bought with his own funds.

ISSUE:

Will the good faith doctrine and applicable Articles in Civil code 1416, In combination with RA Nos.
133, 4381 and Rep. Act No. 4882, the proceeds of the sale would be remitted to him, by way of refund for the
money he used to purchase the said properties. To bar the petitioner from recovering the subject properties,
or at the very least, the money used for the purchase and to bar the petitioner to the proceeds thereof is to
enrich the respondent at his expense, and can this withstand the Constitutional proscription on the property
ownership for the aliens in the Philippines?

Hence, this petition, after successive defeat from lower and appellate court.

HELD.

No. The petition is Denied.

The Court ruled on evidence on record: to wit;

“The three parcels of land subject of the complaint were not mortgaged to the petitioner by the owners
thereof but were sold to the respondent as the vendee, albeit with the use of the petitioners personal funds.”
The sales of three parcels of land in favor of the petitioner who is a foreigner is illegal per se. The
transactions are void ab initio because they were entered into in violation of the Constitution. Thus, to allow
the petitioner to recover the properties or the money used in the purchase of the parcels of land would be
subversive of public policy.

This finds support under Section 7, Article XII of the 1987 Philippine Constitution which states: “Save in cases of
hereditary succession, no private lands shall be transferred or conveyed except to individuals, corporations or
associations qualified to acquire or hold lands of the public domain.”

The court stated: MEMO CUM ALTERIUS DETER DETREMENTO PROTEST (No person should unjustly enrich
himself at the expense of another). An action for recovery of what has been paid without just cause has been
designated as an accion in rem verso.

This provision does not apply if, as in this case, the action is proscribed by the Constitution or by the
application of the pari delicto doctrine.
It may be unfair and unjust to bar the petitioner from filing an accion in rem verso over the subject properties,
or from recovering the money he paid for the said properties, but, as Lord Mansfield stated in the early case
of Holman vs. Johnson. ”The objection that a contract is immoral or illegal as between the plaintiff and the
defendant, sounds at all times very ill in the mouth of the defendant. It is not for his sake, however, that the
objection is ever allowed; but it is founded in general principles of policy, which the defendant has the
advantage of, contrary to the real justice, as between him and the plaintiff, therefore, the petition is bereft of
merits.

53 Muller vs. Muller (GR no. 149615, August 29, 2006) BALUCANAG, April Gem Balansag
FACTS::
Elena Buenaventura Muller and Helmut Muller are husband and wife in this case. They wed and resided in
Germany until they decided to permanently reside in the Philippines in 1992. By this time, they bought a
house in Antipolo, Rizal using the proceeds that they got from selling the house the Helmut Muller inherited
from his parents in Germany.
The marriage, however, did not last. They were eventually separated and Helmut Muller filed for separation
of their properties. The trial court then dissolved the absolute community of property and ordered the
equal partition of their personal properties located within the country, excluding those acquired by
gratuitous title during marriage. As to the Antipolo property, the court held that it was acquired using the
paraphernal funds of Helmut, however, he cannot recover said property, nor have a right to recover the
funds used to buy it since it was purchased in violation of Section 7, Article XII of the Constitution. Upon
appeal, the Court of Appeals granted Helmut Muller’s prayer for reimbursement for the Antipolo property.

ISSUE:
WON Helmut Muller is entitled to reimbursement of the funds used to acquire the Antipolo property.

HELD:
NO. Respondent was aware of the constitutional prohibition and expressly admitted his knowledge thereof
to this Court. He declared that he had the Antipolo property titled in the name of petitioner because of the
said prohibition. His attempt at subsequently asserting or claiming a right on the said property cannot be
sustained. Thus, in the instant case, respondent cannot seek reimbursement on the ground of equity where
it is clear that he willingly and knowingly bought the property despite the constitutional prohibition. To
allow reimbursement would in effect permit respondent to enjoy the fruits of a property which he is not
allowed to own. Thus, it is likewise proscribed by law.

54 Lee vs. Director of Lands (GR No. 128195, October 3, 2001) APURADA, Kathryne Vencio
FACTS::
Sometime in March 1936, Carmen, Francisco, Jr., Ramon, Lourdes, Mercedes, Concepcion, Mariano, Jose,
Loreto, Manuel, Rizal, and Jimmy, all surnamed Dinglasan sold to Lee Liong, A Chinese citizen, a parcel of land
with an approximate area of 1,631 square meters, designed as lot 398 and covered by Original Certificate of
Title No. 3389.
However, in 1948, the former owners filed with the Court of First Instance, Capiz an action against the heirs of
Lee Liong for annulment of sale and recovery of land. The plaintiffs assailed the validity of the sale because of
the constitutional prohibition against aliens acquiring ownership of private agriculture land, including
residential, commercial or industrial land.
Plaintiffs appealed to the Supreme Court and ruled thus: pari delicto (in sales of real estate to aliens incapable
of holding title thereto by virtue of provision of the Constitution, both the vendor and vendee are deemed to
have committed the constitutional violation and thus the courts will not afford protection to either party).
On July 1, 1968, the same former owners (Dinglasans) filed with the Court of First Instance and action for
recovery of the same parcel of land. On Sept. 23, 1968, the heirs of Lee Liong file with the trial. Both cases
were elevated to the Supreme Court but were dismissed holding the suit barred by res judicata. On Sept. 7,
1993, Elizabeth Manuel-Lee and Pacita Yu Lee filed with the RTC, Roxas City a petition for reconstitution of
title of Lot No. 398 of Capiz.
They were the widows of the heirs of Lee Liong, the owner of the lot. Both widows received a parcel of land
through succession from their deceased husbands. RTC ordered the reconstitution of the lost and destroyed
certificate of said title of lot. On Jan. 25, 1995, OSG filed with the Court of Appeals a petition for annulment of
judgment of reconstitution alleging that petitioners were not the proper parties in the reconstitution of title,
since Lee Liong did not acquire title to the lot because he was a Chinese citizen and was constitutionally not
qualified to own the subject land. CA decided, declaring the judgment of reconstitution to be void. Hence this
petition.

Issue/s:
1. WON OSG has the capacity to object the reconstitution.
2. WON constitutional prohibition still exists considering the land is now under the ownership of Lee’s heirs
thru succession.

Held:
1. Yes.
The fact that the Court did not annul the sale of the land t an alien did not validate the transaction. For it was
still contrary to the constitutional proscription against aliens acquiring lands of the public or private domain.
However, the proper party to assail the illegality of the transaction as was not the parties to the transaction.
The proper party to assail the sale is the Solicitor General. This was what was done in this case when the
Solicitor General initiated an action for annulment of judgement of reconstitution of title. While it took the
Republic more than sixty years to assert it, it is not barred from initiating such action. Prescription never lies
against the State.
2. No.
The constitutional proscription on alien ownership of lands of the public or private domain was intended to
protect lands from falling in the hands of non- Filipinos. In this case, however, there would be no more public
policy violated since the land is in the hands of Filipinos qualified to acquire and own such land. “If the land is
invalidly transferred to an alien who subsequently becomes a citizen or transfers it to a citizen, the flaw in the
original transaction is considered cured and the title of the transferee is rendered valid.”
55 Ramirez vs. Vda de Ramirez (111 SCRA 704) ANTOPINA, Babielen Poliquit
Republic vs. Hachero ( G.R. No. 200973, May 30, 2016)
FACTS::
In 1996, Amor Hachero (Hachero) filed his Free Patent Application No. 045307-969 covering Lot No. 1514,
CAD-1150-D (subject land) before the Community Environment and Natural Resources Office (CENRO) of
Palawan. The said application for free patent was later approved by the Provincial Environment and
Natural Resources Officer (PENRO) of Palawan based on the following findings, among others that the land
applied for had been classified as alienable and disposable and, therefore, subject to disposition under the
Public Land Law;
On October 15, 1998, Free Patent No. 045307-98-9384 was issued to Hachero and the subject land was
registered under Original Certificate of Title (OCT) No. E-18011 on May 7, 1999.
After an inspection and verification were conducted by the CENRO in 2000, it was discovered that the
subject land, covered by OCT No. E-18011, was still classified as timberland and so not susceptible of private
ownership under the Free Patent provision of the Public Land Act. Hence, the Republic filed the Complaint
for the Cancellation of Free Patent No. 045307-98-9384 and OCT No. E-18011 and for Reversion, which was
docketed as Civil Case No. 3726.
The RTC rendered its decision in favor of Hachero on the ground that the free patent and title had already
been issued after Hachero was found to have complied with all the requirements; that it was the Republic
itself thru the DENR-CENRO, Coron, which brought the subject land under the operation of the Torrens
System. The CA affirmed the RTC decision.
Issue:
1. Won the petition for cancellation of the title and reversion of the subject lot is meritorious.
2. WON the Government is estopped by its previous acts.
Ruling:
1. Cancellation of title and reversion proper
where there exists a mistake or oversight in
granting free patent over inalienable land
Reversion is an action where the ultimate relief sought is to revert the land back to the government under
the Regalian doctrine. Considering that the land subject of the action originated from a grant by the
government, its cancellation therefore is a matter between the grantor and the grantee. xxx. In Estate of
the Late Jesus S. Yujuico v. Republic (Yujuico case), reversion was defined as an action which seeks to
restore public land fraudulently awarded and disposed of to private individuals or corporations to the mass
of public domain. It bears to point out, though, that the Court also allowed the resort by the Government to
actions for reversion to cancel titles that were void for reasons other than fraud, i.e., violation by the
grantee of a patent of the conditions imposed by law; and lack of jurisdiction of the Director of Lands to
grant a patent covering inalienable forest land or portion of a river, even when such grant was made
through mere oversight.d
In the case at bench, although the Republic's action for cancellation of patent and title and for reversion
was not based on fraud or misrepresentation on the part of Hachero, his title could still be cancelled and the
subject land reverted back to the State because the grant was made through mistake or oversight. This
could probably be the reason why, shortly after one (1) year from the issuance of OCT No. E-18011 to
Hachero, the DENR personnel conducted another investigation and verification on the subject land. It would
appear that they suspected that a mistake was made in their issuance of the patent as the subject land had
not been reclassified or released as alienable or disposable land. It remained plotted within the timberland
classification zone.
2. Prescription and estoppel cannot
lie against the State
At any rate, it is a time-honored principle that the statute of limitations or the lapse of time does not run
against the State. Jurisprudence also recognizes the State's immunity from estoppel as a result of the
mistakes or errors of its officials and agents.
It must be emphasized that a certificate of title issued under an administrative proceeding pursuant to a
homestead patent, as in the instant case, is as indefeasible as a certificate of title issued under a judicial
registration proceeding, provided the land covered by said certificate is a disposable public land within the
contemplation of the Public Land Law. Yet, we emphasize that our statement in the aforequoted case that a
certificate of title issued pursuant to a homestead patent becomes indefeasible after one year, is subject to
the proviso that "the land covered by said certificate is a disposable public land within the contemplation of
the Public Land Law."
But then again, the Court had several times in the past recognized the right of the State to avail itself of the
remedy of reversion in other instances when the title to the land is void for reasons other than having been
secured by fraud or misrepresentation.
Be that as it may, the mistake or error of the officials or agents of the BOL in this regard cannot be invoked
against the government with regard to property of the public domain. It has been said that the State cannot
be estopped by the omission, mistake or error of its officials or agents.
58 Republic vs Umali (GR No. 80687, April 10, 1989) RAÑON, Rexie Monicimpo
FACTS::
The land in question is situated in Tanza, Cavite, and consists of 78,865 square meters. It was originally
purchased on installment from the government on July 1, 1910 by Florentina Bobadilla, who allegedly
transferred her rights thereto in favor of Martina, Tomasa, Gregorio and Julio, all surnamed Cenizal, in
1922. Tomasa and Julio assigned their shares to Martina, Maria and Gregorio. In 1971, these three assignees
purportedly signed a joint affidavit which was filed with the Bureau of Lands to support their claim that they
were entitled to the issuance of a certificate of title over the said land on which they said they had already
made full payment. On the basis of this affidavit, the Secretary of Agriculture and Natural Resources executed
Deed No. V-10910 (Sale Certificate No. 1280) on September 10, 1971, in favor of the said
affiants. Subsequently, on October 13, 1971, TCT No. 55044 (replacing Bobadilla's OCT No. 180) was issued by
the register of deeds of Cavite in favor of Maria Cenizal, Gregorio Cenizal, and (in lieu of Martina Cenizal)
Rosalina Naval, Luz Naval, and Enrique Naval.
When the complaint for reversion was filed on October 10, 1985, the registered owners of the land, following
several transfers, were Remedios Miclat under TCT No. 80392, Juan C. Pulido under TCT No. 80393, and
Rosalina, Luz and Enrique Naval under TCT No. 80394. They were named as defendants and asked to return
the property to the State on the aforestated grounds of forgery and fraud. The plaintiff claimed that Gregorio
Cenizal having died on February 25, 1943, and Maria Cenizal on January 8, 1959, they could not have signed
the joint affidavit dated August 9, 1971, on which Deed No. V-10910 (Sale Certificate No. 1280) was
based. The present holders of the property claiming to be innocent purchasers for value and not privy to the
alleged forgery, contend that the action cannot lie against them.
ISSUE:
WON the sale itself be considered null and void from the start, as the petitioner insists, so as to make all titles
derived therefrom also ineffectual, void ab initio.

HELD:
No. Section 44 of P.D. 1529 (then Sec. 39 of the Land Reg. Act), states that, every registered owner receiving a
certificate of title in pursuance of a decree of registration, and every subsequent purchaser of registered land
taking a certificate of title for value and in good faith, shall hold the same free from all encumbrances except
those noted on the certificate and any of the encumbrances which may be subsisting, and enumerated in the
law. Under said provision, claims and liens of whatever character, except those mentioned by law as existing
against the land prior to the issuance of certificate of title, are cut off by such certificate if not noted thereon,
and the certificate so issued binds the whole world, including the government. The Torrens system was
adopted in this country because it was believed to be the most effective measure to guarantee the integrity of
land titles and to protect their indefeasibility once the claim of ownership is established and recognized. If a
person purchases a piece of land on the assurance that the seller’s title thereto is valid, he should not run the
risk of being told later that his acquisition was ineffectual after all.
We find that the private respondents are transferees in good faith and for value of the subject property and
that the original acquisition thereof, although fraudulent, did not affect their own titles. These are valid
against the whole world, including the government.
59 Piñero vs Director of Lands (GR No. L-36507, 12/14/1974) PLAZA, Mariafe Manatad
60 Kionisala vs. Dacut (GR No. 147379, 2/27/2002) PELAEZ, Chester Bryan Pepito
61 Evangelista vs. Santiago (GR No. 157447, 4/29/2005) PASAOL, Ric Jason Patlingrao
62 Roco vs. Gimeda (GR No. L-11651, 12/27/1958) PAMISARAN, Excel Joy Gemota
63 Republic v. Hachero, G.R. No. 200973, [May 30, 2016]) NALLA, Glene Alacayde
64 Republic v. Espinosa, G.R. No. 186603, [April 5, 2017]) MISTERIO, John Kessler Sumauang
FACTS:
A cadastral decree was issued in favor of Espinosa. The Original Certificate of Title was issued in the name of
Espinosa who later sold to Caliston which a Transfer Certificate of Title was issued.

The State through Regional Executive Director of the DENR filed a Complaint for annulment of title and
reversion of land with the RTC claiming the property is inalienable public land because it fell within the
timberland area.

RTC ruled in favor of the State and ordered reversion of the property.

CA ruled in favor of Espinosa and found that the State failed to prove fraud or misrepresentation when she
was issued the Original Certificate of Title. It further ruled that the State failed to prove that the property is
forest land. The lone piece of evidence consisting of LC Map No, 2978 was not authenticated pursuant to
Section 24 Rule 132 of the Rules of Court. It noted that the parties stipulated only as to the existence of the
map, but not as to genuineness of truthfulness of its content. Assuming that the map is admitted in
evidence, Espinosa’s rights over the property, which accrued in 1962, should not be prejudiced by a
subsequent classification by the State done in 1986, or after 24 years.

Issue
1st Issue: Whether or not the State has sufficiently proved that the property is part of inalienable forest land
at the time Espinosa was granted the cadastral decree and issued a title.

2nd Issue: WON the State may initiate an action for Reversion over the subject land.
Held
No. The State failed to prove that the property was classified as forest land at the time of the grant of the
cadastral decree and issuance of title to Espinosa.

Since cadastral proceedings are governed by the usual rules of practice, procedure, and evidence, a
cadastral decree and a certificate of title are issued only after the applicant proves all the requisite
jurisdictional FACTS:-that they are entitled to the claimed lot, that all parties are heard, and that evidence is
considered.31 As such, the cadastral decree is a judgment which adjudicates ownership after proving these
jurisdictional FACTS:.

Here, it is undisputed that Espinosa was granted a cadastral decree and was subsequently issued OCT No.
191-N, the predecessor title of Caliston’s TCT No. 91117.

Having been granted a decree in a cadastral proceeding, Espinosa can be presumed to have overcome the
presumption that the land sought to be registered forms part of the public domain.33 This means that
Espinosa, as the applicant, was able to prove by incontrovertible evidence that the property is alienable and
disposable property in the cadastral proceedings.
This is not to say, however, that the State has no remedy to recover the property if indeed it is part of the
inalienable lands of the public domain. The State may still do so through an action for reversion, as in the
present case.

2nd Issue: WON the State may initiate an action for Reversion over the subject land.

No.

Reversion is the remedy where the State, pursuant to the Regalian doctrine, seeks to revert land back to the
mass of the public domain. It is proper when public land is fraudulently awarded and disposed of to private
individuals or corporations.

There are also instances when we granted reversion on grounds other than fraud, such as when a “person
obtains a title under the Public Land Act which includes, by oversight, lands which cannot be registered under
the Torrens system, or when the Director of Lands did not have jurisdiction over the same because it is of the
public domain.”36

In this case, the State, through the Solicitor General, alleges neither fraud nor misrepresentation in the
cadastral proceedings and in the issuance of the title in Espinosa’s favor. No mistake or oversight was also
found in the issuance of title. Hence, the remedy of reversion could not be applied in this case.

You might also like